Annotation of db/baza/spb15.txt, revision 1.2

1.1       rubashki    1: Чемпионат:
                      2: XVII Чемпионат Санкт-Петербурга (2015)
                      3: 
                      4: Дата:
                      5: 04-Oct-2015 - 01-Nov-2015
                      6: 
                      7: Тур:
1 тур. "Ла Скала"
                      8: 
                      9: Дата:
                     10: 04-Oct-2015
                     11: 
                     12: Вопрос 1:
                     13: В 2015 году президент Обама совершил поездку на Аляску. Сара Пэйлин,
                     14: отметив, что президент ограничился туристическими целями, а не
                     15: воспользовался случаем публично пригрозить соседям по шельфу, поставила
                     16: ему в пример президента Рузвельта и сообщила, что ему стоило бы взять с
                     17: собой ПЕРВУЮ, а не ВТОРУЮ. Назовите и ПЕРВУЮ, и ВТОРУЮ.
                     18: 
                     19: Ответ:
                     20: Большая дубинка, палка для селфи.
                     21: 
                     22: Зачет:
                     23: Big stick, selfie stick.
                     24: 
                     25: Комментарий:
                     26: На экскурсии президент, по теперешней моде, сделал несколько селфи.
                     27: Пэйлин намекала на политику "большой дубинки" (Big Stick Policy),
                     28: предложенную президентом Теодором Рузвельтом.
                     29: 
                     30: Источник:
                     31:    1. http://www.washingtontimes.com/news/2015/sep/6/sarah-palin-ex-alaska-governor-calls-on-obama-to-c/
                     32:    2. http://lenta.ru/news/2015/09/07/palin_criticising_obama/
                     33:    3. http://ru.wikipedia.org/wiki/Политика_большой_дубинки
                     34: 
                     35: Автор:
                     36: Александр Мартынов
                     37: 
                     38: Вопрос 2:
                     39: Как известно, победителем конкурса "Имя Россия" стал Александр Невский.
                     40: Победителем аналогичного конкурса "Великие румыны" стал господарь Стефан
                     41: Великий, чей полный титул звучит как Штефан чел Маре ши Сфынт.
                     42: Переведите слово "сфынт" на русский язык.
                     43: 
                     44: Ответ:
                     45: Святой.
                     46: 
                     47: Комментарий:
                     48: Как и Александр Невский.
                     49: 
                     50: Источник:
                     51:    1. http://ru.wikipedia.org/wiki/Стефан_III_Великий
                     52:    2. http://ru.wikipedia.org/wiki/100_величайших_румын
                     53:    3. http://ru.wikipedia.org/wiki/Имя_Россия
                     54: 
                     55: Автор:
                     56: Артем Москаленко
                     57: 
                     58: Вопрос 3:
                     59: (pic: 20150700.jpg)
                     60:    Художник Куанг Хай рисует от руки супергероев, дополняя изображения
                     61: объектами реального мира. Мы не спрашиваем, как он назвал супергероя с
                     62: розданной вам картинки. Ответьте максимально точно, что на ней мы
                     63: закрыли белым кругом.
                     64: 
                     65: Ответ:
                     66: Бальзам "Золотая звезда".
                     67: 
                     68: Зачет:
                     69: Бальзам "Звездочка"; Као Шао Ванг.
                     70: 
                     71: Комментарий:
                     72: (pic: 20150701.jpg)
                     73:    Куанг Хай - художник из Вьетнама - назвал супергероя Капитан Вьетнам.
                     74: В руке у нового супергероя баночка бальзама "Золотая звезда", внешне
                     75: напоминающая вибраниумовый щит супергероя Капитана Америки из вселенной
                     76: Marvel.
                     77: 
                     78: Источник:
                     79: http://thehz.ru/interes/supergeroi-dopolnennye-okruzhayushhimi-obektami
                     80: 
                     81: Автор:
                     82: Павел Уточкин
                     83: 
                     84: Вопрос 4:
                     85: Внимание, вопрос четвертый!
                     86:    В вопросе словами "ПСЫ" и "РЫЦАРИ" заменены другие слова.
                     87:    Способ, который некоторое время назад стали применять в Финляндии для
                     88: снижения аварийности на трассах, заставляет самих финнов называть своих
                     89: ПСОВ РЫЦАРЯМИ. Мы же в связи с этим способом, по мнению журналиста,
                     90: скорее вспомним другого сэра. Какое слово мы заменили на "РЫЦАРИ"?
                     91: 
                     92: Ответ:
                     93: Джедаи.
                     94: 
                     95: Комментарий:
                     96: (pic: 20150702.jpg)
                     97:    Одна из распространенных причин аварий в Финляндии - столкновения с
                     98: оленями, выходящими на дороги. Для того чтобы сделать оленей заметнее,
                     99: их рога начали красить светоотражающей краской, что напоминает финнам
                    100: джедаев со световыми мечами. По мнению корреспондентки, при упоминании
                    101: светящегося ночью животного уместнее ассоциация с Проклятием
                    102: Баскервилей. Ну и да, вопрос четвертый. May the 4th be with you. :-)
                    103: 
                    104: Источник:
                    105: https://vk.com/1tvnews?w=wall-49388814_104897
                    106: 
                    107: Автор:
                    108: Анастасия Мартынова
                    109: 
                    110: Вопрос 5:
                    111: Запишите строчку из стихотворения Владимира Маяковского, написанного в
                    112: 1920-х годах: "Я считаю: оба плохи". Комсомолец - герой этого
                    113: стихотворения - критикует двух людей за то, что они охраняют короля.
                    114: Один публицист использовал данное стихотворение как аргумент в вопросе о
                    115: том, как правильно... Как правильно делать что?
                    116: 
                    117: Ответ:
                    118: Писать фамилию Алёхин.
                    119: 
                    120: Зачет:
                    121: Произносить фамилию Алёхин.
                    122: 
                    123: Комментарий:
                    124: "Я считаю: оба плохи - Капабланка и Алёхин. Оба-два, в игре юля,
                    125: охраняли короля". Известны споры, "е" или "ё" писать в фамилии
                    126: четвертого чемпиона мира. Шахматный публицист Геннадий Сосонко считает,
                    127: что стихотворение современника может служить определенным аргументом в
                    128: пользу защиты буквы "ё".
                    129: 
                    130: Источник:
                    131:    1. В.В. Маяковский. Нагрузка по макушку.
                    132: http://www.stihi-xix-xx-vekov.ru/m-stih369.html
                    133:    2. http://www.chess-news.ru/node/13653
                    134: 
                    135: Автор:
                    136: Александр Мартынов
                    137: 
                    138: Вопрос 6:
                    139: Сент-Люсия - небольшое государство, находящееся на архипелаге Малые
                    140: Антильские острова. Оно не имеет больших традиций в беговых дисциплинах,
                    141: однако однажды во время крупного спортивного форума сразу пять атлетов
                    142: из Сент-Люсии оказались на стадионе раньше всех остальных. В каком
                    143: городе это произошло?
                    144: 
                    145: Ответ:
                    146: Афины.
                    147: 
                    148: Комментарий:
                    149: Αγία
                    150: Λουκία ("АгИа ЛукИа"), т.е.
                    151: "Святая Луция", - так звучит название Сент-Люсии по-гречески (ср.
                    152: агиология, агиография и т.д.), поэтому ее название шло первым по
                    153: алфавиту. По традиции олимпийские парады в разных странах мира открывает
                    154: команда Греции, затем следуют команды по алфавиту, а последней - команда
                    155: хозяев. Однако на домашней Олимпиаде Греция шла последней. Обилие слов
                    156: греческого происхождения в вопросе - небольшая подсказка.
                    157: 
                    158: Источник:
                    159:    1. http://ru.wikipedia.org/wiki/Церемония_открытия_летних_Олимпийских_игр_2004
                    160:    2. http://www.youtube.com/watch?v=FuJ_zTmToLs&t=46m50s
                    161: 
                    162: Автор:
                    163: Павел Уточкин
                    164: 
                    165: Вопрос 7:
                    166: (pic: 20150703.jpg) (pic: 20150704.jpg)
                    167:    Чтобы продемонстрировать, насколько благородный и горделивый народ
                    168: темные эльфы, создатели фильма "Тор-2: Царство тьмы" намеренно занизили
                    169: ЭТО. Назовите ЭТО тремя словами.
                    170: 
                    171: Ответ:
                    172: Прорези для глаз.
                    173: 
                    174: Комментарий:
                    175: Для эльфов были придуманы специальные маски с заниженными прорезями для
                    176: глаз. Чтобы видеть через них, актерам приходилось держать спину ровно, а
                    177: голову запрокидывать, что придавало темным эльфам надменный вид.
                    178: 
                    179: Источник:
                    180: Буклет "Формула кино", N 11 (51), ноябрь 2013 г. - С. 39.
                    181: 
                    182: Автор:
                    183: Анастасия Мартынова
                    184: 
                    185: Вопрос 8:
                    186:    <раздатка>
                    187:    Charles Calthrop
                    188:    </раздатка>
                    189:    Чарльз Колтроп, герой одного романа, планирует совершить теракт во
                    190: Франции. Напишите прозвище, под которым он фигурирует в романе.
                    191: 
                    192: Ответ:
                    193: Шакал.
                    194: 
                    195: Комментарий:
                    196: Если соединить первые три буквы имени и первые три буквы фамилии, то
                    197: получится слово "Chacal" - "шакал" на французском языке. Кстати, теракты
                    198: во Франции совершал и другой Шакал - террорист Карлос Рамирес Санчес.
                    199: 
                    200: Источник:
                    201: Ф. Форсайт. День Шакала.
                    202: 
                    203: Автор:
                    204: Александр Мартынов
                    205: 
                    206: Вопрос 9:
                    207: Рассуждая о том, как его изменила эмиграция, голландский шахматист Генна
                    208: Сосонко говорит, что вывезенный из Советского Союза Сосонко походил на
                    209: своего нынешнего тезку разве что ИМИ. ЕГО проверяют, в частности, на
                    210: устойчивость к механическим повреждениям при броске с высоты трех и
                    211: десяти метров. Назовите ЕГО.
                    212: 
                    213: Ответ:
                    214: Спасательный круг.
                    215: 
                    216: Комментарий:
                    217: Спасательный круг должен выдерживать бросок на воду с высоты десяти
                    218: метров и на землю с высоты трех метров. Автор говорит: "Сосонко,
                    219: которого я вывез из Советского Союза, похож на своего тезку Sosonko
                    220: разве что тремя спасательными кругами буквы "о"".
                    221: 
                    222: Источник:
                    223:    1. Г. Сосонко. Тогда. Шахматные эссе.
                    224:    2. http://ru.wikipedia.org/wiki/Спасательный_круг
                    225: 
                    226: Автор:
                    227: Анастасия Мартынова
                    228: 
                    229: Вопрос 10:
                    230: Некоторые модели мощных советских зенитных ракет пилоты называли
                    231: "летающими ИМИ". Интернет-пользователь Sarmis [сармис] шутливо отмечает,
                    232: что в Древней Греции ИХ, очевидно, не было, несмотря на древнегреческие
                    233: корни. Ведь иначе подвиг Фидиппида не понадобился бы. Назовите ИХ двумя
                    234: словами.
                    235: 
                    236: Ответ:
                    237: Телеграфные столбы.
                    238: 
                    239: Комментарий:
                    240: Американцы называли советские ракеты за большие размеры "летающими
                    241: телеграфными столбами". Слово "телеграф" происходит от двух
                    242: древнегреческих корней (&tau;&#8134;&lambda;&epsilon; - "далеко" +
                    243: &gamma;&rho;&#8112;&phi;&omega; - "пишу"). Существование телеграфных
                    244: столбов подразумевает наличие телеграфа, с помощью которого сообщение о
                    245: победе при Марафоне могло бы быть передано без самопожертвования героя.
                    246: 
                    247: Источник:
                    248:    1. http://www.defence.gov.au/news/raafnews/editions/5305/5305.pdf
                    249: ("Almost every ground-borne threat an aircrew can face is displayed,
                    250: from 12.7mm heavy machine guns through to 'Flying Telegraph Pole' SA-2
                    251: surface-to-air missiles.")
                    252:    2. http://ru.wikipedia.org/wiki/Дикие_ласки
                    253:    3. http://searchengines.guru/showthread.php?t=318834
                    254: 
                    255: Автор:
                    256: Павел Уточкин
                    257: 
                    258: Вопрос 11:
                    259: [Двухсторонняя раздатка.
                    260:    На верхней стороне, видной командам, напечатано:
                    261:    По нашему мнению, команды, которые соблюдают Кодекс и не смотрят
                    262: раздаточный материал до сигнала ведущего, выступают в турнирах более
                    263: успешно.
                    264:    На обратной стороне напечатано:
                    265:    Профессор Стэнфордского университета писал, что первоначально почти
                    266: разуверился в своей теории, поскольку его не воспринимали всерьез и
                    267: заявку на грант для исследований предложили переадресовать АЛЬФЕ. В
                    268: слогане фильма, в названии которого фигурирует разновидность АЛЬФЫ, есть
                    269: слова "Приготовьтесь ко вкусу приключений". Назовите наиболее известную
                    270: разработку этого профессора.]
                    271:    В вопросе есть замена.
                    272:    Профессор Стэнфордского университета писал, что первоначально почти
                    273: разуверился в своей теории, поскольку его не воспринимали всерьез и
                    274: заявку на грант для исследований предложили переадресовать АЛЬФЕ. В
                    275: слогане фильма, в названии которого фигурирует разновидность АЛЬФЫ, есть
                    276: слова "Приготовьтесь ко вкусу приключений". Назовите наиболее известную
                    277: разработку этого профессора.
                    278: 
                    279: Ответ:
                    280: Маршмеллоу-тест.
                    281: 
                    282: Зачет:
                    283: Зефирный тест и прочие синонимичные ответы.
                    284: 
                    285: Комментарий:
                    286: Уолтер Мишел - разработчик метода под названием "Парадигма
                    287: самостоятельного отказа дошкольников от получения немедленного
                    288: удовлетворения ради отсроченных более ценных вознаграждений". В качестве
                    289: исследования силы воли детям предлагали сладости, вдвое большее
                    290: количество которых можно было получить, дождавшись ассистента. Первую
                    291: заявку на грант отклонили, "послав" автора на кондитерскую фабрику.
                    292: Фильм, о котором идет речь в вопросе, - "Чарли и шоколадная фабрика".
                    293: 
                    294: Источник:
                    295:    1. У. Мишел. Развитие силы воли. - М.: "Манн, Иванов и Фербер", 2015.
                    296:    2. http://www.kinonews.ru/movie_131/charlie-and-the-chocolate-factory
                    297: 
                    298: Автор:
                    299: Анастасия Мартынова
                    300: 
                    301: Вопрос 12:
                    302: Летом 1996 года в Элисте прошел матч за звание чемпиона мира по
                    303: шахматам. Статья о матче в журнале "64" отмечала, что ОН уверенно
                    304: победил в состязании: сначала отложил партию с шансами на выигрыш, а
                    305: затем успешно реализовал материальный перевес. Напишите его фамилию.
                    306: 
                    307: Ответ:
                    308: Ельцин.
                    309: 
                    310: Комментарий:
                    311: Одновременно с матчем проходили и выборы президента России. Победивший в
                    312: матче Анатолий Карпов уподобил первый тур, по итогам которого Ельцин
                    313: лидировал, но не набрал 50%, откладыванию партии с шансами на выигрыш, а
                    314: второй тур - доигрыванию. Сам же матч состоял, разумеется, более чем из
                    315: одной партии.
                    316: 
                    317: Источник:
                    318: Журнал "64", 1996, N 7. - С. 3.
                    319: 
                    320: Автор:
                    321: Александр Мартынов
                    322: 
                    323: Вопрос 13:
                    324: Ботаники Жан-Поль Виньерон и Виржини Лусс, фотографируя это растение в
                    325: разных лучах спектра, обнаружили, что оно почти полностью поглощает
                    326: ультрафиолет, способный обжечь листья. Его изображение встречается еще
                    327: на надгробии римского легионера первого века нашей эры, найденном в
                    328: Каринтии. Назовите это растение.
                    329: 
                    330: Ответ:
                    331: Эдельвейс.
                    332: 
                    333: Комментарий:
                    334: Воздействие ультрафиолета на растения особенно сильно в горах, по
                    335: причине разреженной атмосферы. Каринтия - альпийская федеральная земля в
                    336: Австрии. Римский легионер принадлежал к горной когорте. Название
                    337: "Эдельвейс" и эмблему в виде цветка в дальнейшем носили ряд
                    338: горно-стрелковых подразделений Германии и других стран.
                    339: 
                    340: Источник:
                    341:    1. http://ru.wikipedia.org/wiki/Эдельвейс
                    342:    2. https://commons.wikimedia.org/wiki/File:CohMont1.jpg
                    343:    3. http://ru.wikipedia.org/wiki/Эдельвейс_(значения)
                    344: 
                    345: Автор:
                    346: Павел Уточкин
                    347: 
                    348: Вопрос 14:
                    349: В вопросе есть замена.
                    350:    Владимир Орлов рассказывал, как для него в детстве предметом особого
                    351: ожидания были ИКСЫ, "преподававшие" географию Родины, а затем и Европы.
                    352: Изобразившего ИКС Джеймса Уистлера раскритиковали за то, что он "просил
                    353: 200 гиней за право выплеснуть банку краски в лицо зрителя". Назовите
                    354: ИКС.
                    355: 
                    356: Ответ:
                    357: Салют. Незачет: Фейерверк.
                    358: 
                    359: Комментарий:
                    360: Салютовали в честь освобожденных городов. Джеймс Уистлер на своей
                    361: картине "Ноктюрн в черном и золотом. Падающая ракета" изобразил
                    362: украсившее небо яркие фейерверки, но Джону Рёскину картина сильно не
                    363: понравилась. Победные салюты давались не фейерверками, так что первой
                    364: части вопроса этот ответ не удовлетворяет.
                    365: 
                    366: Источник:
                    367:    1. В.В. Орлов. И наступило - "после войны".
                    368: http://www.flibusta.net/b/40997/read
                    369:    2. http://www.baikalski.com/library/history_of_paints_27.html
                    370: 
                    371: Автор:
                    372: Анастасия Мартынова
                    373: 
                    374: Вопрос 15:
                    375:    <раздатка>
                    376:    В 1960-х годах театр на Таганке поставил знаменитую пьесу Бертольда
                    377: Брехта под названием "Добрый человек из Сезуана". Под каким названием
                    378: она была впервые поставлена одним российским театром 12 сентября 2013
                    379: года?
                    380:    </раздатка>
                    381:    Мы дополнительно информируем команды, что команда, перевернувшая
                    382: раздатку до сигнала ведущего, будет дисквалифицирована.
                    383:    Можете переворачивать раздатку. Вопрос перед вами. Время.
                    384: 
                    385: Ответ:
                    386: "Последнее китайское предупреждение".
                    387: 
                    388: Комментарий:
                    389: В оригинале речь о Сычуани - провинции в Китае. Наше предупреждение о
                    390: дисквалификации по действенности тоже похоже на последнее китайское. Да
                    391: и вопрос последний.
                    392: 
                    393: Источник:
                    394: http://www.mtfontanka.spb.ru/repertuar/tekushchij-repertuar/item/113-poslednee-k
                    395: 
                    396: Автор:
                    397: Александр Мартынов
                    398: 
                    399: Тур:
2 тур. "Дикие бозоны Хиггса"
                    400: 
                    401: Дата:
                    402: 04-Oct-2015
                    403: 
                    404: Инфо:
                    405: Хотелось бы поблагодарить за помощь в работе: Яну Азриэль, Елену
                    406: Анциферову, Владимира Городецкого, Аркадия Илларионова, Андрея
                    407: Кокуленко, Артема Колесова, Сергея Лобачёва, Константина Науменко,
                    408: Дмитрия Овчарука, Владимира Островского, Ивана Семушина, Ульяну
                    409: Фабричнину.
                    410: 
                    411: Вопрос 1:
                    412: Дуплет.
                    413:    1. ОН вышел в 2004 году. Тогда же ОН вошел в число номинантов на
                    414: "Оскар". Назовите ЕГО.
                    415:    2. ОНИ идут с 2010 года. В 2011 году ОНИ вошли в число номинантов на
                    416: "Золотой глобус". Назовите ИХ.
                    417: 
                    418: Ответ:
                    419:    1. "Ходячий замок".
                    420:    2. "Ходячие мертвецы".
                    421: 
                    422: Комментарий:
                    423: Многократно повторяемое слово "идти" в разных формах - подсказка. В
                    424: первом вопросе речь идет о мультфильме Хаяо Миядзаки, во втором - об
                    425: известном телесериале о последствиях зомби-апокалипсиса.
                    426: 
                    427: Источник:
                    428:    1. http://ru.wikipedia.org/wiki/Ходячий_замок_(аниме)
                    429:    2. http://ru.wikipedia.org/wiki/Ходячие_мертвецы_(телесериал)
                    430: 
                    431: Автор:
                    432: Николай Коврижных
                    433: 
                    434: Вопрос 2:
                    435: (pic: 20150705.jpg)
                    436:    Перед вами пример росписи вазы, применявшейся в переходный период
                    437: древнегреческого искусства. Такие вазы, называемые словом "ДИГЛОССА",
                    438: сравнивают с находкой XVIII века. Какое слово мы заменили словом
                    439: "ДИГЛОССА"?
                    440: 
                    441: Ответ:
                    442: Билингва.
                    443: 
                    444: Комментарий:
                    445: Один и тот же сюжет на вазе изображен в краснофигурном и чернофигурном
                    446: варианте. Такие вазы похожи на Розеттский камень, найденный в 1799 году
                    447: в Египте, на котором надпись была выполнена на трех языках. "Диглосса" -
                    448: это калька слова "билингва" на греческий язык.
                    449: 
                    450: Источник:
                    451:    1. http://ru.wikipedia.org/wiki/Билингва_(вазопись)
                    452:    2. Сравнение автора вопроса.
                    453: 
                    454: Автор:
                    455: Михаил Рыжок
                    456: 
                    457: Вопрос 3:
                    458: Малая пагода диких гусей в Сиане несколько раз уцелела во время
                    459: разрушительных землетрясений. Современные исследования показали, что
                    460: пагода стоит на каменной глыбе, благодаря чему ее можно сравнить с
                    461: головой ЭТОГО. Назовите ЭТО, используя дефис.
                    462: 
                    463: Ответ:
                    464: Ванька-встанька.
                    465: 
                    466: Источник:
                    467: http://ru.wikipedia.org/wiki/Малая_пагода_диких_гусей
                    468: 
                    469: Автор:
                    470: Михаил Рыжок
                    471: 
                    472: Вопрос 4:
                    473: В рассказе Конан Дойля конный разведчик, увидев два дома, стоящие друг
                    474: напротив друга, догадался о существовании... Чего?
                    475: 
                    476: Ответ:
                    477: Брода.
                    478: 
                    479: Комментарий:
                    480: Увидев два дома, стоящие друг напротив друга на противоположных берегах
                    481: реки, разведчик решил, что существует брод, по которому их жители ходят
                    482: друг к другу.
                    483: 
                    484: Источник:
                    485: А. Конан Дойл. Подвиги бригадира Жерара.
                    486: http://www.flibusta.net/b/214587/read
                    487: 
                    488: Автор:
                    489: Михаил Рыжок
                    490: 
                    491: Вопрос 5:
                    492: В одном стихотворении Наталья Меркушова называет слезы своим капиталом.
                    493: Символично, что глагол, которым начинается стихотворение, стоит в форме
                    494: первого лица в настоящем времени. Напишите этот глагол.
                    495: 
                    496: Ответ:
                    497: "ПлачУ".
                    498: 
                    499: Комментарий:
                    500: Сравнивая свои слезы с монетами, автор подобрала удачный глагол: "Плачу
                    501: я звонкою монетой, ее чеканю я из слез".
                    502: 
                    503: Источник:
                    504: http://www.merkushova-ns.ru/part/Novoe/26
                    505: 
                    506: Автор:
                    507: Николай Коврижных
                    508: 
                    509: Вопрос 6:
                    510: Патриция Бойд сравнила свою работу с обнаружением блохи на автомобильной
                    511: фаре. Проект, в котором занята Бойд, получил имя в честь немца. Напишите
                    512: фамилию этого немца.
                    513: 
                    514: Ответ:
                    515: Кеплер.
                    516: 
                    517: Комментарий:
                    518: Телескоп "Кеплер", запущенный NASA, занимается поиском экзопланет
                    519: транзитным методом - по уменьшению света звезды при прохождении перед
                    520: ней планеты. Тень от планеты, заслоняющей звезду, можно сравнить с
                    521: блохой, ползущей по автомобильной фаре.
                    522: 
                    523: Источник:
                    524: "Популярная механика", 2009, N 6. - С. 17.
                    525: 
                    526: Автор:
                    527: Михаил Рыжок
                    528: 
                    529: Вопрос 7:
                    530: (pic: 20150706.jpg)
                    531:    Искусствовед Юрий Кузнецов сравнил тарелку на натюрморте Виллема
                    532: Кальфа с чашей ЕГО. Какой художник нарисовал более полусотни ИХ?
                    533: 
                    534: Ответ:
                    535: [Винсент] ван Гог.
                    536: 
                    537: Комментарий:
                    538: Тарелка с фруктами расположена на натюрморте ориентированной на свет и
                    539: поэтому сравнивается с чашей подсолнуха. Кисти ван Гога принадлежат две
                    540: серии картин с подсолнухами.
                    541: 
                    542: Источник:
                    543:    1. http://www.bibliotekar.ru/kholland/8.htm
                    544:    2. https://www.hermitagemuseum.org/wps/portal/hermitage/digital-collection/01.+Paintings/44878/?lng=ru
                    545:    3. http://ru.wikipedia.org/wiki/Подсолнухи_(картина_Ван_Гога)
                    546: 
                    547: Автор:
                    548: Михаил Рыжок
                    549: 
                    550: Вопрос 8:
                    551: По одной из версий, ОНИ произошли от одичавших бэренбейцеров, завезенных
                    552: викингами. Игорь Акимушкин рассказывает легенду о том, как один из НИХ
                    553: спас жизнь Наполеону, когда тот отплывал с острова Эльба. Назовите ИХ
                    554: словом русского или словом английского происхождения.
                    555: 
                    556: Ответ:
                    557: Водолазы.
                    558: 
                    559: Зачет:
                    560: Ньюфаундленды.
                    561: 
                    562: Комментарий:
                    563: Бэренбейцеры - старая порода собак, которыми травили медведей. Легенда
                    564: гласит, что водолаз вытащил из воды Наполеона, когда тот нечаянно выпал
                    565: из лодки при отплытии.
                    566: 
                    567: Источник:
                    568: И.И. Акимушкин. Мир животных. Том 6. Домашние животные.
                    569: http://www.flibusta.net/b/376610/read
                    570: 
                    571: Автор:
                    572: Михаил Рыжок
                    573: 
                    574: Вопрос 9:
                    575: Солдат из романа Конан Дойля поднимает тост за тис, коноплю и за НИХ.
                    576: Какому инструменту ОНО дало название?
                    577: 
                    578: Ответ:
                    579: Перочинному ножу.
                    580: 
                    581: Комментарий:
                    582: Старый лучник поднимает тост за тис, из которого делается древко лука,
                    583: за коноплю, из которого вьется тетива, и за гусиные перья, служащие для
                    584: стрел. Гусиные перья также использовались для письма, и нож для их
                    585: очинки получил название перочинного.
                    586: 
                    587: Источник:
                    588: А. Конан Дойл. Белый отряд. http://www.flibusta.net/b/273255/read
                    589: 
                    590: Автор:
                    591: Михаил Рыжок
                    592: 
                    593: Вопрос 10:
                    594: (pic: 20150707.jpg)
                    595:    На этом пародийном рисунке мы скрыли от вас ИХ. Монумент в городе
                    596: Мирный представляет собой ИХ, символизирующие рода войск. Назовите ИХ
                    597: двумя словами.
                    598: 
                    599: Ответ:
                    600: Три меча.
                    601: 
                    602: Комментарий:
                    603: Композиция на розданном рисунке напоминает композицию картины Жака Луи
                    604: Давида "Клятва Горациев". Монумент в городе Мирный представляет собой
                    605: воткнутые в землю мечи, символизирующие войска на земле, в небесах и на
                    606: море.
                    607: 
                    608: Источник:
                    609:    1. http://hiero.ru/2198419
                    610:    2. http://www.arctic-info.ru/Regions/196/RegionLife/03-04-2015/monyment-tri-meca
                    611: 
                    612: Автор:
                    613: Наталья Горелова
                    614: 
                    615: Вопрос 11:
                    616: Герою Джорджа Мартина, разбуженному заревом ночного пожара, кажется, что
                    617: ПРОПУСК. ПРОПУСК и для гипотетического наблюдателя на Венере. Заполните
                    618: пропуск четырьмя словами.
                    619: 
                    620: Ответ:
                    621: Солнце восходит на западе.
                    622: 
                    623: Комментарий:
                    624: Спросонья герой принимает далекое зарево за восход Солнца и удивляется,
                    625: почему Солнце восходит на западе, а не на востоке. Венера и Уран
                    626: вращаются в направлении, противоположном направлению большинства планет,
                    627: поэтому Солнце там действительно должно восходить на западе.
                    628: 
                    629: Источник:
                    630:    1. Дж.Р.Р. Мартин. Присяжный рыцарь.
                    631:    2. http://ru.wikipedia.org/wiki/Венера
                    632: 
                    633: Автор:
                    634: Михаил Рыжок
                    635: 
                    636: Вопрос 12:
                    637: Чтобы обезопасить себя, ОНИ выбросили оружие. О произведении,
                    638: посвященном ИМ, в романе "Молчание ягнят" размышляет Ганнибал Лектер.
                    639: Назовите это произведение.
                    640: 
                    641: Ответ:
                    642: "Плот "Медузы"".
                    643: 
                    644: Комментарий:
                    645: Ганнибал Лектер известен своим пристрастием к человеческому мясу. Этим
                    646: же пристрастием пришлось проникнуться и пассажирам плота "Медузы" за
                    647: долгие дни, проведенные в открытом океане. Под конец несколько выживших
                    648: выбросили всё оружие, чтобы не перерезать друг друга.
                    649: 
                    650: Источник:
                    651:    1. http://arzamas.academy/episodes/65
                    652:    2. Т. Харрис. Молчание ягнят. http://www.flibusta.net/b/392309/read
                    653: 
                    654: Автор:
                    655: Наталья Горелова
                    656: 
                    657: Вопрос 13:
                    658: Братья Вайнеры сравнили с НЕЙ кукурузу, которую очень аккуратно ела
                    659: девушка. В каком произведении ОНА помогает пройти сквозь огонь и воду?
                    660: 
                    661: Ответ:
                    662: "Волшебная флейта".
                    663: 
                    664: Комментарий:
                    665: Героиня держит кукурузу двумя руками и откусывает деликатно, будто
                    666: собирается играть на флейте. В опере Моцарта "Волшебная флейта"
                    667: благодаря этой самой флейте происходит несколько чудес - на то она и
                    668: волшебная.
                    669: 
                    670: Источник:
                    671:    1. А.А. Вайнер, Г.А. Вайнер. Эра милосердия.
                    672: http://www.flibusta.net/b/324990/read
                    673:    2. http://ru.wikipedia.org/wiki/Волшебная_флейта
                    674: 
                    675: Автор:
                    676: Михаил Рыжок
                    677: 
                    678: Вопрос 14:
                    679:    <раздатка>
                    680:    Всё растет круглый год, в этом убедишься ты,
                    681:    Да, легко, как ... : ... ... ...
                    682:    </раздатка>
                    683:    Децл был популярен в конце 90-х. В одной из своих песен он говорит,
                    684: что домашнее садоводство не требует больших усилий, и приглашает
                    685: заниматься им. Закончите строчку.
                    686: 
                    687: Ответ:
                    688: "Инвайт: просто добавь воды".
                    689: 
                    690: Комментарий:
                    691: "Инвайт", как и Децл, - привет из девяностых. Инвайт - это приглашение.
                    692: 
                    693: Источник:
                    694: Le Truk, "Шкаф".
                    695: 
                    696: Автор:
                    697: Николай Коврижных
                    698: 
                    699: Вопрос 15:
                    700: С 2008 года СДЕЛАТЬ ЭТО можно в одном месте, воспользовавшись двумя
                    701: лестницами. С конца мая текущего года можно СДЕЛАТЬ ЭТО в другом месте,
                    702: воспользовавшись пятью полотнами. Ответьте как можно точнее: что такое
                    703: "СДЕЛАТЬ ЭТО"?
                    704: 
                    705: Ответ:
                    706: Попасть с пятой линии на первую.
                    707: 
                    708: Зачет:
                    709: По упоминанию попадания или перехода с пятой линии на первую.
                    710: 
                    711: Комментарий:
                    712: В первом случае речь о переходе со станции "Звенигородская" на
                    713: "Пушкинскую", во втором - о переходе со "Спортивной" на Первую линию
                    714: Васильевского острова. Для этого понадобится проехать на трех
                    715: эскалаторах и двух траволаторах.
                    716: 
                    717: Источник:
                    718: ЛОАВ.
                    719: 
                    720: Автор:
                    721: Николай Коврижных
                    722: 
                    723: Тур:
3 тур. "Рабочее название"
                    724: 
                    725: Дата:
                    726: 04-Oct-2015
                    727: 
                    728: Инфо:
                    729: Команда благодарит за помощь в подготовке тура Владислава Говердовского
                    730: и Викторию Мелехсон.
                    731: 
                    732: Вопрос 1:
                    733: В декабре 2011 года Гвидо Тонелли сделал важное заявление, в котором
                    734: фигурировал ИКС. Какие ИКСЫ идут вторыми в списке, опубликованном в
                    735: сентябре 2015 года?
                    736: 
                    737: Ответ:
                    738: Дикие.
                    739: 
                    740: Комментарий:
                    741: Гвидо Тонелли - один из главных авторов открытия бозонов Хиггса в
                    742: Большом адронном коллайдере.
                    743: 
                    744: Источник:
                    745:    1. http://en.wikipedia.org/wiki/Guido_Tonelli
                    746:    2. http://chgk-spb.livejournal.com/726962.html
                    747: 
                    748: Автор:
                    749: Юрий Выменец
                    750: 
                    751: Вопрос 2:
                    752: В кабинах фуникулёра в Бильбао рядом с обычным запрещающим знаком
                    753: расположен почти такой же, на который добавлены плюс и минус. Ответьте
                    754: точно, что запрещает делать этот дополнительный знак.
                    755: 
                    756: Ответ:
                    757: Курить электронные сигареты.
                    758: 
                    759: Комментарий:
                    760: (pic: 20150708.jpg)
                    761:    В кабинах фуникулёра нельзя курить даже электронные сигареты. Знак
                    762: очень похож, но на изображение электронной сигареты добавлены с разных
                    763: сторон плюс и минус.
                    764: 
                    765: Источник:
                    766:    1. ЛОАВ.
                    767:    2. http://www.danaandjeffluxuryhomes.com/calabasas-bans-e-cigs-wherever-regular-cigarettes-are-prohibited/
                    768: 
                    769: Автор:
                    770: Виктория Мелехсон
                    771: 
                    772: Вопрос 3:
                    773: В финале американского фильма говорится, что героя убили не аэропланы, а
                    774: красота. Назовите этого героя.
                    775: 
                    776: Ответ:
                    777: Кинг-Конг.
                    778: 
                    779: Комментарий:
                    780: "It wasn't the airplanes. It was Beauty that killed the Beast".
                    781: 
                    782: Источник:
                    783: http://www.youtube.com/watch?v=zw3O30xGTvE
                    784: 
                    785: Автор:
                    786: Владислав Говердовский
                    787: 
                    788: Вопрос 4:
                    789: В повести Гоголя описана картина, на которой грешники, освобождённые в
                    790: день Страшного Суда, гоняют чёрта этими предметами. От этого предмета,
                    791: по-видимому, происходит фамилия художника. Назовите этот предмет.
                    792: 
                    793: Ответ:
                    794: Полено.
                    795: 
                    796: Источник:
                    797: Н.В. Гоголь. Ночь перед Рождеством. http://www.flibusta.net/b/74286/read
                    798: 
                    799: Автор:
                    800: Юрий Выменец
                    801: 
                    802: Вопрос 5:
                    803: Персонаж Стругацких встречает инопланетянина, похожего на огромного
                    804: паука. Вечером персонаж вспоминает встречу, задумчиво глядя на ЭТО.
                    805: Назовите ЭТО двумя словами, начинающимися на одну и ту же букву.
                    806: 
                    807: Ответ:
                    808: Созвездие Скорпиона.
                    809: 
                    810: Комментарий:
                    811: Скорпионы относятся к классу паукообразных. Вечером после наступления
                    812: темноты можно увидеть звёзды.
                    813: 
                    814: Источник:
                    815: А.Н. Стругацкий, Б.Н. Стругацкий. Извне.
                    816: http://www.flibusta.net/b/93847/read
                    817: 
                    818: Автор:
                    819: Юрий Выменец
                    820: 
                    821: Вопрос 6:
                    822: Сейсмолог Луиджи Пикарди отмечает, что через НЕГО проходит тектонический
                    823: разлом под названием Great Glen, что вызывает частые землетрясения и
                    824: выделения газов. Назовите ЕГО.
                    825: 
                    826: Ответ:
                    827: [Озеро] Лох-Несс.
                    828: 
                    829: Комментарий:
                    830: Ещё одно объяснение чудовища.
                    831: 
                    832: Источник:
                    833: http://blogs.scientificamerican.com/history-of-geology/the-earth-shattering-loch-ness-monster-that-wasnt/
                    834: 
                    835: Автор:
                    836: Сергей Николенко, Ирина Прокофьева
                    837: 
                    838: Вопрос 7:
                    839: ЕЁ можно определить по-разному. Например, "то, популярность чего резко
                    840: снизилась в США после инаугурационной речи моложавого Джона Кеннеди".
                    841: Назовите ЕЁ.
                    842: 
                    843: Ответ:
                    844: Шляпа.
                    845: 
                    846: Комментарий:
                    847: Считается, что у Кеннеди были красивые волосы, и президент любил их
                    848: демонстрировать. Он был первым президентом, появившимся на инаугурации
                    849: без шляпы. Вопрос намекает на игру "Шляпа".
                    850: 
                    851: Источник:
                    852: К. Маккей, Б. Маккей. Искусство быть настоящим мужчиной. Классические
                    853: навыки и манеры для современных мужчин.
                    854: 
                    855: Автор:
                    856: Владислав Говердовский, Юрий Выменец
                    857: 
                    858: Вопрос 8:
                    859: В 1963 году Индонезия официально объявила, что собирается удлинить ЕГО
                    860: название на четыре буквы. Назовите ЕГО.
                    861: 
                    862: Ответ:
                    863: Индийский океан.
                    864: 
                    865: Комментарий:
                    866: Индонезия решила переименовать его в Индонезийский. :-) Не прижилось.
                    867: 
                    868: Источник:
                    869: D.W. Fryer. Emerging Southeast Asia: A Study in Growth and Stagnation.
                    870: New York, Wiley, 1979, p. 6.
                    871: 
                    872: Автор:
                    873: Сергей Николенко
                    874: 
                    875: Вопрос 9:
                    876: В одном из рассказов О. Генри описывается картина, на самом ТАКОМ -
                    877: вернее, СЯКОМ - месте которой высилось великолепное венецианское
                    878: палаццо. Слова "ТАКОЙ" и "СЯКОЙ" получаются друг из друга заменой одной
                    879: буквы на другую. Напишите оба слова.
                    880: 
                    881: Ответ:
                    882: Видный, водный.
                    883: 
                    884: Источник:
                    885: О. Генри. Чародейные хлебцы. http://www.flibusta.net/b/432959/read
                    886: 
                    887: Автор:
                    888: Ирина Прокофьева
                    889: 
                    890: Вопрос 10:
                    891: Знаете ли вы что-нибудь об императоре Священной Римской империи Карле
                    892: VII? Фридрих Великий охарактеризовал его, заменив в известном латинском
                    893: выражении логический оператор. Как именно охарактеризовал?
                    894: 
                    895: Ответ:
                    896: И Цезарь, и никто.
                    897: 
                    898: Зачет:
                    899: Et Caesar et nihil; Цезарь и никто; Caesar et nihil.
                    900: 
                    901: Источник:
                    902: Е.В. Анисимов. Елизавета Петровна. http://www.flibusta.net/b/176689/read
                    903: 
                    904: Автор:
                    905: Александр Либер
                    906: 
                    907: Вопрос 11:
                    908: Этот итальянский учёный создал специальный аппарат для окрашивания
                    909: отдельных нервов и клеток, что позволило существенно продвинуться в
                    910: понимании строения клетки. Назовите его.
                    911: 
                    912: Ответ:
                    913: Гольджи.
                    914: 
                    915: Комментарий:
                    916: Позже в честь Гольджи назвали совсем другой аппарат.
                    917: 
                    918: Источник:
                    919: http://en.wikipedia.org/wiki/Camillo_Golgi
                    920: 
                    921: Автор:
                    922: Сергей Николенко
                    923: 
                    924: Вопрос 12:
                    925: Альпинисты жалуются на то, что восхождение на Эверест стало слишком
                    926: популярным; на некоторых этапах подъёма людям приходится даже стоять в
                    927: длинной очереди. Альпинист Грэм Хойланд сравнивает Эверест с НИМ. Одно
                    928: из ЕГО предложений названо в честь другой горы. Какой?
                    929: 
                    930: Ответ:
                    931: МакКинли.
                    932: 
                    933: Зачет:
                    934: Денали.
                    935: 
                    936: Комментарий:
                    937: Хойланд сравнивает Эверест с "Макдональдсом".
                    938: 
                    939: Источник:
                    940:    1. http://www.bbc.com/russian/society/2013/05/130528_everest_crowds
                    941:    2. http://www.hackthemenu.com/mcdonalds/secret-menu/mckinley-mac/
                    942: 
                    943: Автор:
                    944: Виктория Мелехсон, Юрий Выменец
                    945: 
                    946: Вопрос 13:
                    947: Картина "Четыре стихии", изображающая четыре аллегорические фигуры, не
                    948: нравилась Альберту Шпееру. Шпеер говорил, что картина должна называться
                    949: "Четыре ПАЛЬЦА", поясняя при этом, что не хватает МИЗИНЦА. Какие слова
                    950: мы заменили словами "ПАЛЕЦ" и "МИЗИНЕЦ"?
                    951: 
                    952: Ответ:
                    953: Чувство, вкус.
                    954: 
                    955: Источник:
                    956: http://saiferfan.livejournal.com/4166.html
                    957: 
                    958: Автор:
                    959: Александр Либер
                    960: 
                    961: Вопрос 14:
                    962: 31 июля 1965 года - самая ранняя дата, на которую когда-либо публично
                    963: указывало это произведение. Назовите это произведение.
                    964: 
                    965: Ответ:
                    966: "Yesterday".
                    967: 
                    968: Комментарий:
                    969: 1 августа песня была впервые исполнена на телешоу "Blackpool Night Out".
                    970: 
                    971: Источник:
                    972: http://ru.wikipedia.org/wiki/Yesterday
                    973: 
                    974: Автор:
                    975: Сергей Николенко
                    976: 
                    977: Вопрос 15:
                    978: Национальный символ страны пришлось развернуть в противоположную
                    979: сторону, чтобы не нарушать правила использования товарного знака,
                    980: зарегистрированного в 1876 году. Какой компанией?
                    981: 
                    982: Ответ:
                    983: "Гиннесс".
                    984: 
                    985: Зачет:
                    986: "Guinness".
                    987: 
                    988: Комментарий:
                    989: Когда Ирландия в 1920-х годах выбирала национальный символ, арфу
                    990: пришлось развернуть: "Гиннесс" зарегистрировала торговый знак за 50 лет
                    991: до основания независимого государства.
                    992: 
                    993: Источник:
                    994: http://www.ndsmcobserver.com/2015/04/chicago-art-institute-presents-ireland/
                    995: 
                    996: Автор:
                    997: Николай Черницкий, Владислав Говердовский
                    998: 
                    999: Тур:
4 тур. "Лимпопо"
                   1000: 
                   1001: Дата:
                   1002: 11-Oct-2015
                   1003: 
                   1004: Инфо:
                   1005: Команда благодарит за тестирование и ценные замечания Антона Тахтарова
                   1006: (Самара), Дмитрия Когана (Имменштад), Галину Воловник (Рыбница), Игоря
                   1007: Тюнькина (Москва), Олега Рахмилевича (Николаев), Максима Мерзлякова,
                   1008: Юлию Мещерякову, Александра Татаринцева, Аркадия Илларионова (все -
                   1009: Воронеж), Ивана Фролова (Москва), Наиля Фарукшина (Навои).
                   1010: 
                   1011: Вопрос 1:
                   1012: ЕЕ сестры взяли себе псевдонимы: ОльгА ВарЕн, ОдИль ВерсуА и ЭлЕн ВальЕ.
                   1013: Назовите ЕЕ.
                   1014: 
                   1015: Ответ:
                   1016: Марина Влади.
                   1017: 
                   1018: Зачет:
                   1019: Екатерина Марина Владимировна Полякова-Байдарова.
                   1020: 
                   1021: Комментарий:
                   1022: Как и ее сестры, она взяла псевдоним, начинающийся на букву V.
                   1023: 
                   1024: Источник:
                   1025: http://ru.wikipedia.org/wiki/Влади,_Марина
                   1026: 
                   1027: Автор:
                   1028: Наталия Рыжанова
                   1029: 
                   1030: Вопрос 2:
                   1031:    <раздатка>
                   1032: |   Oct 20   Manchester U  1-2  Bolton
                   1033: |   Nov 3    Liverpool     3-1  Manchester U
                   1034: |   Nov 24   Arsenal       3-1  Manchester U
                   1035: |   Sep 15   Newcastle     4-3  Manchester U
                   1036: |   Dec 1    Manchester U  0-3  Chelsea
                   1037:    </раздатка>
                   1038:    Сезон 2001/02 годов "Манчестер Юнайтед" начал неудачно. К декабрю
                   1039: было проиграно пять матчей. Назовите уроженца города АлЕс, которого
                   1040: болельщики считали одним из главных виновников неудач.
                   1041: 
                   1042: Ответ:
                   1043: [Лоран] Блан.
                   1044: 
                   1045: Зачет:
                   1046: Blanc.
                   1047: 
                   1048: Комментарий:
                   1049: Болельщики обратили внимание, что из первых букв команд, обыгравших
                   1050: "Манчестер Юнайтед", получается фамилия французского защитника.
                   1051: 
                   1052: Источник:
                   1053:    1. http://www.sports.ru/tribuna/blogs/innuendo/475668.html
                   1054:    2. http://www.rsssf.com/tablese/eng02.html
                   1055:    3. http://ru.wikipedia.org/wiki/Блан,_Лоран
                   1056: 
                   1057: Автор:
                   1058: Александр Колышкин
                   1059: 
                   1060: Вопрос 3:
                   1061: В телевизионном сюжете о биатлонисте Устюгове говорилось, что его
                   1062: супруга сразу после свадьбы оставила спорт и стала мужниным талисманом.
                   1063: Какое слово в вопросе мы заменили?
                   1064: 
                   1065: Ответ:
                   1066: Жениным.
                   1067: 
                   1068: Комментарий:
                   1069: Устюгова зовут Женя, жена стала Жениным талисманом.
                   1070: 
                   1071: Источник:
                   1072: "В новый год с олимпийскими чемпионами", канал "Россия-2", 01.01.2015 г.
                   1073: 
                   1074: Автор:
                   1075: Наталия Рыжанова
                   1076: 
                   1077: Вопрос 4:
                   1078:    <раздатка>
                   1079:    Она ПРОПУСК
                   1080:    Сумела прожевать,
                   1081:    Но тут ей рот заткнула
                   1082:    Двуспальная кровать.
                   1083:    </раздатка>
                   1084:    Перед вами окончание шуточного стихотворения, в котором
                   1085: рассказывается, как Уильям Мёрфи повстречался с акулой. Мы не просим
                   1086: заполнить пропуск. Назовите псевдоним, под которым выступал Уильям
                   1087: Мёрфи.
                   1088: 
                   1089: Ответ:
                   1090: [Силач] [Сальваторе] Бамбула.
                   1091: 
                   1092: Зачет:
                   1093: Макс Бамбула.
                   1094: 
                   1095: Комментарий:
                   1096: Уильям Мёрфи в начале XX века выступал на аренах цирка под псевдонимом
                   1097: Силач Бамбула. Рассказывают, что однажды шпрехшталмейстер одесского
                   1098: цирка пошутил: "Силач Бамбула поднял четыре стула...". Потом эта
                   1099: дразнилка в разных вариантах разлетелась по городам и весям.
                   1100: 
                   1101: Источник:
                   1102:    1. http://oldatletikaussr.blogspot.ru/2012/11/blog-post_6520.html
                   1103:    2. http://www.wrestlingdata.com/index.php?befehl=bios&wrestler=24101
                   1104: 
                   1105: Автор:
                   1106: Александр Колышкин
                   1107: 
                   1108: Вопрос 5:
                   1109: Внимание, в вопросе есть замены.
                   1110:    В 1536 году Луис ИКС отказался от обещанных его деду привилегий,
                   1111: получив при этом пенсию и титулы герцога Верагуа и маркиза Ямайки. Эту
                   1112: сделку можно назвать "ИКСОВ ИГРЕК". Википедия в статье "ИКСОВ ИГРЕК"
                   1113: упоминает, среди прочего, лошадь, арбуз, индейку и картофель. Что мы
                   1114: заменили словами "ИКСОВ ИГРЕК"?
                   1115: 
                   1116: Ответ:
                   1117: Колумбов обмен.
                   1118: 
                   1119: Комментарий:
                   1120: Внук Христофора Колумба отказался от притязаний на Панамский перешеек.
                   1121: "Колумбов обмен" - термин, обозначающий перемещение большого количества
                   1122: растений, животных, технологий и культурных достижений, а также групп
                   1123: населения из Старого Света в Новый и наоборот.
                   1124: 
                   1125: Источник:
                   1126:    1. http://en.wikipedia.org/wiki/Luis_Col%C3%B3n,_1st_Duke_of_Veragua
                   1127:    2. http://ru.wikipedia.org/wiki/Колон,_Диего
                   1128:    3. http://ru.wikipedia.org/wiki/Колумбов_обмен
                   1129: 
                   1130: Автор:
                   1131: Александр Колышкин
                   1132: 
                   1133: Вопрос 6:
                   1134: В этом вопросе слово "АЛЬФА" заменяет другое слово.
                   1135:    В одном сериале выпускник Оксфорда упоминает девушек, управляющих
                   1136: лодками по-кембриджски. На самом деле он имеет в виду АЛЬФ. Назовите
                   1137: европейский регион, жители которого считают подобное употребление слова
                   1138: "АЛЬФА" недопустимым.
                   1139: 
                   1140: Ответ:
                   1141: Лесбос.
                   1142: 
                   1143: Комментарий:
                   1144: Такие лодки называются панты. В Кембридже и Оксфорде пантами принято
                   1145: управлять по-разному. В Кембридже принято управлять лодкой, стоя на
                   1146: корме, а в Оксфорде - на носу. Оксфордец имеет в виду женщину с
                   1147: нетрадиционной ориентацией.
                   1148: 
                   1149: Источник:
                   1150:    1. Телесериал "Lost in Austen", 2-я серия, примерно 3-я минута.
                   1151:    2. http://www.imdb.com/title/tt1290007/trivia?item=tr0667082
                   1152:    3. http://ru.wikipedia.org/wiki/Лесбиянство
                   1153: 
                   1154: Автор:
                   1155: Мария Колышкина, Александр Колышкин
                   1156: 
                   1157: Вопрос 7:
                   1158: После победы над англичанами в Плимутском сражении голландский адмирал
                   1159: Михаил де Рёйтер получил прозвище "Ушастый тюлень". Немецкий адмирал
                   1160: Эрих Редер сыграл заметную роль в том, что "Ушастый тюлень" так и не был
                   1161: реализован. Какие два слова мы заменили словами "Ушастый тюлень"?
                   1162: 
                   1163: Ответ:
                   1164: Морской лев.
                   1165: 
                   1166: Комментарий:
                   1167: Плимутское сражение было морским. Адмирал Редер был одним из тех, кто
                   1168: отговорил Гитлера от высадки в Британии (операция "Морской лев").
                   1169: Морские львы относятся к семейству ушастые тюлени.
                   1170: 
                   1171: Источник:
                   1172:    1. http://ru.wikipedia.org/wiki/Плимутское_сражение
                   1173:    2. http://ru.wikipedia.org/wiki/Операция_%C2%ABМорской_лев%C2%BB
                   1174:    3. http://ru.wikipedia.org/wiki/Морские_львы
                   1175: 
                   1176: Автор:
                   1177: Александр Колышкин
                   1178: 
                   1179: Вопрос 8:
                   1180: В заметке об одном пабе говорится, что с его террасы можно посмеяться
                   1181: над неумехами на реке Кем. В заметке упоминается "ПОРЦИЯ с видом на
                   1182: ПЕРЦЫ". Какие слова мы заменили ПОРЦИЕЙ и ПЕРЦАМИ?
                   1183: 
                   1184: Ответ:
                   1185: Пинта, панты.
                   1186: 
                   1187: Комментарий:
                   1188: Про панты вы могли узнать из комментария к одному из предыдущих
                   1189: вопросов.
                   1190: 
                   1191: Источник:
                   1192:    1. http://www.forbes.ru/guide/velikobritaniya/69552-kembridzh
                   1193:    2. http://www.ru-uk.ru/panting-v-kembridzhe/
                   1194: 
                   1195: Автор:
                   1196: Мария Колышкина, Александр Колышкин
                   1197: 
                   1198: Вопрос 9:
                   1199: Уроженец Иллинойса Фредерик Маклафлин служил в дивизии, названной в
                   1200: честь индейского вождя Макатавимешекаки. Переведите имя Макатавимешекаки
                   1201: на английский или русский язык.
                   1202: 
                   1203: Ответ:
                   1204: Be a large black hawk.
                   1205: 
                   1206: Зачет:
                   1207: Black Hawk; блэк хок; черный ястреб.
                   1208: 
                   1209: Комментарий:
                   1210: Маклафлин - основатель хоккейного клуба "Чикаго Блэкхокс". Он служил в
                   1211: 86-й пехотной дивизии "Черный ястреб" и назвал клуб в честь нее.
                   1212: 
                   1213: Источник:
                   1214:    1. http://en.wikipedia.org/wiki/Frederic_McLaughlin
                   1215:    2. http://en.wikipedia.org/wiki/Black_Hawk_(Sauk_leader)
                   1216:    3. http://ru.wikipedia.org/wiki/Чикаго_Блэкхокс
                   1217: 
                   1218: Автор:
                   1219: Александр Колышкин
                   1220: 
                   1221: Вопрос 10:
                   1222: Мод Сильвер, героиня произведений Патриции Вентворт, периодически пишет
                   1223: письма своей подруге, в которых просит совета. Ответы она получает, как
                   1224: правило, в конце произведения. Назовите фамилию подруги.
                   1225: 
                   1226: Ответ:
                   1227: Марпл.
                   1228: 
                   1229: Комментарий:
                   1230: Мисс Мод Сильвер - частный детектив в произведениях Патриции Вентворт.
                   1231: Любопытно, что она впервые появилась в книге раньше, чем мисс Марпл
                   1232: Агаты Кристи. У Патриции Вентворт старые дамы дружат, и периодически
                   1233: мисс Марпл дает ценные советы подруге. Так, например, в произведении
                   1234: "Китайская шаль", после того как мисс Сильвер разоблачила преступника,
                   1235: она получает письмо от мисс Марпл, в котором рассказывается об
                   1236: аналогичном случае.
                   1237: 
                   1238: Источник:
                   1239:    1. http://ru.wikipedia.org/wiki/Патриция_Вентворт
                   1240:    2. П. Вентворт. Китайская шаль. http://www.flibusta.net/b/58967/read
                   1241: 
                   1242: Автор:
                   1243: Александр Колышкин
                   1244: 
                   1245: Вопрос 11:
                   1246: В современной миниатюре одна средневековая дама обладает несдержанным
                   1247: характером и склонностью к экстравагантным выходкам. А вынужденный
                   1248: выгораживать ее супруг жалуется, что войдет в историю как подкаблучник и
                   1249: извращенец с плохим чувством юмора. Назовите эту даму.
                   1250: 
                   1251: Ответ:
                   1252: [Леди] Годива.
                   1253: 
                   1254: Комментарий:
                   1255: Леди Годива напивается и скачет голышом по Ковентри. Чтобы замять это
                   1256: происшествие, граф "задним числом" объявляет, что обещал сократить
                   1257: налоги, если жена сделает это. При этом жалуется, что войдет в историю
                   1258: не лучшим образом.
                   1259: 
                   1260: Источник:
                   1261:    1. http://www.liveinternet.ru/users/rianon-n/post304309467/
                   1262:    2. http://ru.wikipedia.org/wiki/Леди_Годива
                   1263: 
                   1264: Автор:
                   1265: Александр Колышкин
                   1266: 
                   1267: Вопрос 12:
                   1268: В американском фильме муж и жена подходят к делу с разных сторон.
                   1269: Разгневанный муж говорит жене, что порвет ее на кусочки и скормит ИМ.
                   1270: Кому ИМ?
                   1271: 
                   1272: Ответ:
                   1273: Присяжным.
                   1274: 
                   1275: Комментарий:
                   1276: Муж - помощник прокурора, а его жена - адвокат. Они должны выступать на
                   1277: одном судебном процессе, что разозлило героя. Слово "разгневанный"
                   1278: отсылает к фильму о присяжных "12 разгневанных мужчин".
                   1279: 
                   1280: Источник:
                   1281: Фильм "Ребро Адама" (1949), реж. Джордж Кьюкор, 33-я минута.
                   1282: 
                   1283: Автор:
                   1284: Наталия Рыжанова
                   1285: 
                   1286: Вопрос 13:
                   1287: Место погребения грузинской царицы Тамары доподлинно неизвестно.
                   1288: Возможно, поэтому один источник утверждает, что царица ДЕЛАЕТ ЭТО.
                   1289: Назовите современника Тамары, который, согласно аналогичному источнику,
                   1290: ДЕЛАЕТ ЭТО к востоку от Гарца.
                   1291: 
                   1292: Ответ:
                   1293: Фридрих I Барбаросса.
                   1294: 
                   1295: Зачет:
                   1296: Барбаросса; Фридрих I Гогенштауфен.
                   1297: 
                   1298: Комментарий:
                   1299: ДЕЛАЕТ ЭТО - спит. По преданию, царица Тамара спит на золотом ложе и,
                   1300: согласно легенде, когда дойдет до нее голос людской скорби, она
                   1301: проснется и воцарится вновь. Барбаросса спит в пещере под горой и,
                   1302: согласно аналогичной легенде, однажды проснется на радость немцам.
                   1303: 
                   1304: Источник:
                   1305:    1. http://ingushetia.rusplt.ru/index/ishut_zahoronenie_tsaritsy_Tamary-10021.html
                   1306:    2. http://ru.wikipedia.org/wiki/Кифхойзер
                   1307:    3. http://ru.wikipedia.org/wiki/Фридрих_I_Барбаросса
                   1308: 
                   1309: Автор:
                   1310: Александр Колышкин
                   1311: 
                   1312: Вопрос 14:
                   1313: Осенью 1918 года после распада Австро-Венгрии в Будапеште началась
                   1314: "революция австрийцев". В одно из слов предыдущего предложения мы
                   1315: добавили несколько букв. Напишите это слово в первоначальном виде.
                   1316: 
                   1317: Ответ:
                   1318: астр.
                   1319: 
                   1320: Комментарий:
                   1321: Цветочные названия революций - вполне обычное дело, астры - осенние
                   1322: цветы.
                   1323: 
                   1324: Источник:
                   1325: http://ru.wikipedia.org/wiki/Революция_астр
                   1326: 
                   1327: Автор:
                   1328: Наталия Рыжанова
                   1329: 
                   1330: Вопрос 15:
                   1331:    <раздатка>
                   1332:    Рекламирующий тариф на мобильную связь для путешественников слоган
                   1333: повторяет текст известного закона без одного слова. Напишите это слово.
                   1334:    </раздатка>
                   1335:    Слоган, рекламирующий тариф на мобильную связь для путешественников,
                   1336: повторяет текст известного закона без одного слова. Напишите это слово.
                   1337: 
                   1338: Ответ:
                   1339: Слагаемых.
                   1340: 
                   1341: Комментарий:
                   1342: Слоган компании "Мегафон" звучал так: "От перемены мест сумма не
                   1343: изменяется". Вы могли заметить, что на раздаточном материале места
                   1344: некоторых слов изменены. "От перемены мест слагаемых сумма не меняется"
                   1345: - это переместительный закон сложения.
                   1346: 
                   1347: Источник:
                   1348: ЛОАВ.
                   1349: 
                   1350: Автор:
                   1351: Сергей Пономарёв
                   1352: 
                   1353: Тур:
5 тур. "ЮМА"
                   1354: 
                   1355: Дата:
                   1356: 11-Oct-2015
                   1357: 
                   1358: Вопрос 1:
                   1359: Этим вопросом мы открываем тур!
                   1360:    В 2000 году в ЮАР появилась улица с таким названием. В 2002 году на
                   1361: ней, помимо остальных, поселилась Ками - "девушка", зараженная ВИЧ.
                   1362: Напишите название этой улицы.
                   1363: 
                   1364: Ответ:
                   1365: Сезам.
                   1366: 
                   1367: Комментарий:
                   1368: В программах, аналогичных "Улице Сезам", в странах Африки есть персонаж
                   1369: Ками - маппет с ВИЧ для образования детей в этой широкораспространенной
                   1370: там проблеме.
                   1371: 
                   1372: Источник:
                   1373: http://en.wikipedia.org/wiki/Kami_(Takalani_Sesame)
                   1374: 
                   1375: Автор:
                   1376: Алексей Шпильман
                   1377: 
                   1378: Вопрос 2:
                   1379: Статья об участии Бенедикта Камбербэтча в шекспировской постановке была
1.2     ! rubashki 1380: озаглавлена следующим образом: "ПРОПУСК1 or not ПРОПУСК2". Восстановите
        !          1381: любой из пропусков.
1.1       rubashki 1382: 
                   1383: Ответ:
                   1384: 221B.
                   1385: 
                   1386: Комментарий:
                   1387: Бенедикт Камбербэтч, известный ролью Шерлока Холмса, сыграл Гамлета в
                   1388: постановке Барбикан-Центра.
                   1389: 
                   1390: Источник:
                   1391: http://www.sherlockshome.net/2015/08/05/221b-or-not-221b-a-comparison-of-sherlock-and-hamlet/
                   1392: 
                   1393: Автор:
                   1394: Жанна Подоляк
                   1395: 
                   1396: Вопрос 3:
                   1397: В 1956 году инженер Уильям Шокли основал в Маунтин-Вью компанию, которая
                   1398: использовала более дешевый материал, чем принятый тогда германий.
                   1399: Принято считать, что именно эта новаторская идея через некоторое время
                   1400: привела к появлению топонима. Какого?
                   1401: 
                   1402: Ответ:
                   1403: Кремниевая долина.
                   1404: 
                   1405: Зачет:
                   1406: Силиконовая долина.
                   1407: 
                   1408: Источник:
                   1409: У. Айзексон. Стив Джобс.
                   1410: https://books.google.ru/books?id=xSWOMjMSzi8C&pg=PT28#v=onepage&q&f=false
                   1411: 
                   1412: Автор:
                   1413: Михаил Скипский
                   1414: 
                   1415: Вопрос 4:
                   1416: До революции ЕЕ называли "золотой пряжкой на стальном поясе России".
                   1417: Назовите ЕЕ четырьмя буквами.
                   1418: 
                   1419: Ответ:
                   1420: КБЖД.
                   1421: 
                   1422: Комментарий:
                   1423: Кругобайкальская железная дорога. До революции этот короткий участок
                   1424: пути называли "золотой пряжкой стального пояса России". Пряжкой - потому
                   1425: что она соединила разорванную Байкалом Транссибирскую магистраль, а
                   1426: золотой - потому что по стоимости работ она превзошла все существующие в
                   1427: России дороги.
                   1428: 
                   1429: Источник:
                   1430: http://www.krugobaikalka.ru/info.php
                   1431: 
                   1432: Автор:
                   1433: Михаил Скипский
                   1434: 
                   1435: Вопрос 5:
                   1436: Режиссер Жан-Люк Годар говорил, что кино - это ОНА двадцать четыре кадра
                   1437: в секунду, а американская актриса Лили Томлин парадоксально считает, что
                   1438: ОНА - это лучшее средство для промывки мозгов. Назовите ЕЕ.
                   1439: 
                   1440: Ответ:
                   1441: Правда.
                   1442: 
                   1443: Источник:
                   1444:    1. http://ru.wikiquote.org/wiki/Жан-Люк_Годар
                   1445:    2. http://ru.wikiquote.org/wiki/Лили_Томлин
                   1446: 
                   1447: Автор:
                   1448: Михаил Скипский
                   1449: 
                   1450: Вопрос 6:
                   1451: Максим Поташев спросил у автора вопроса, как пишется "АЛЬФА". В
                   1452: английском городе Рединг находится центр по изучению АЛЬФЫ. Что мы
                   1453: заменили на АЛЬФУ?
                   1454: 
                   1455: Ответ:
                   1456: Дислексия.
                   1457: 
                   1458: Комментарий:
                   1459: Поташев не знал, как пишется это слово, - что само по себе является
                   1460: одним из симптомов дислексии. Reading - по-английски "чтение", что
                   1461: делает этот факт довольно смешным.
                   1462: 
                   1463: Источник:
                   1464:    1. http://www.dyslexic.org.uk
                   1465:    2. http://ru.wikipedia.org/wiki/Дислексия
                   1466: 
                   1467: Автор:
                   1468: Алексей Шпильман
                   1469: 
                   1470: Вопрос 7:
                   1471: В городе Сансет-Вэлли - одном из крупных городов вселенной игры "The
                   1472: Sims 3" - на кладбище стоит в том числе и ЕЕ статуя. Что нужно сделать с
                   1473: НЕЙ в этой игре, чтобы спасти умершего персонажа?
                   1474: 
                   1475: Ответ:
                   1476: Обыграть в шахматы.
                   1477: 
                   1478: Комментарий:
                   1479: ОНА - это Смерть, и в игре присутствует явная аллюзия на фильм "Седьмая
                   1480: печать".
                   1481: 
                   1482: Источник:
                   1483:    1. http://ru.sims.wikia.com/wiki/Сансет_Вэлли
                   1484:    2. http://ru.sims.wikia.com/wiki/Костлявая_Смерть
                   1485: 
                   1486: Автор:
                   1487: Михаил Скипский
                   1488: 
                   1489: Вопрос 8:
                   1490: Фильм, в котором ОН впервые появился на киноэкране, вышел в 1973 году, а
                   1491: ОН почему-то был отрицательным персонажем, противостоявшим Капитану
                   1492: Америке. Фильм "ОНА" вышел в 1944 году и имел детективный сюжет.
                   1493: Назовите ЕЕ по-английски.
                   1494: 
                   1495: Ответ:
                   1496: Spider Woman.
                   1497: 
                   1498: Комментарий:
                   1499: ОН - разумеется, spiderman.
                   1500: 
                   1501: Источник:
                   1502:    1. http://ru.wikipedia.org/wiki/Паучиха
                   1503:    2. http://ru.wikipedia.org/wiki/Человек-паук#.D0.9A.D0.B8.D0.BD.D0.BE_.D0.B8_.D1.82.D0.B5.D0.B0.D1.82.D1.80
                   1504: 
                   1505: Автор:
                   1506: Михаил Скипский
                   1507: 
                   1508: Вопрос 9:
                   1509: В статье Википедии "АЛЬФА" упоминаются ООН и УЕФА. Система "АЛЬФА"
                   1510: применяется на военных судах США для защиты от ракет противника. Какое
                   1511: слово мы заменили на АЛЬФУ?
                   1512: 
                   1513: Ответ:
                   1514: Эгида.
                   1515: 
                   1516: Комментарий:
                   1517: Эгидой назывался щит Зевса (по некоторым мифам - Афины). Целый ряд
                   1518: мероприятий проходит под эгидой ООН и УЕФА.
                   1519: 
                   1520: Источник:
                   1521:    1. http://ru.wikipedia.org/wiki/Эгида
                   1522:    2. http://en.wikipedia.org/wiki/Aegis_Combat_System
                   1523: 
                   1524: Автор:
                   1525: Александр Мосягин
                   1526: 
                   1527: Вопрос 10:
                   1528: ЕГО автобиография называется "В Земле Гигантов", а умение пользоваться
                   1529: своим ростом, для того чтобы "украсть" мяч, закрепило за НИМ его
                   1530: прозвище. Назовите ЕГО фамилию.
                   1531: 
                   1532: Ответ:
                   1533: Боггз.
                   1534: 
                   1535: Комментарий:
                   1536: Тайрон Куртис "Магси" Богз (Tyrone Curtis "Muggsy" Bogues) - одна из
                   1537: звезд НБА 1990-х с невероятно маленьким для баскетбола ростом 160 см.
                   1538: Прозвище "Muggsy" - уменьшительное от английского "Mugger" - грабитель,
                   1539: воришка.
                   1540: 
                   1541: Источник:
                   1542: http://en.wikipedia.org/wiki/Muggsy_Bogues
                   1543: 
                   1544: Автор:
                   1545: Михаил Скипский
                   1546: 
                   1547: Вопрос 11:
                   1548: ОНИ появились в 1994 году. У НИХ есть три "глаза", и ОНИ могут "выжить",
                   1549: даже если от НИХ останется всего 70%. В Японии ИХ можно встретить на
                   1550: надгробных плитах. Назовите ИХ.
                   1551: 
                   1552: Ответ:
                   1553: QR-коды.
                   1554: 
                   1555: Источник:
                   1556:    1. http://ru.wikipedia.org/wiki/QR-код
                   1557:    2. http://www.livemaster.ru/topic/269145-krasivyj-dizajn-dlya-qr-koda
                   1558: 
                   1559: Автор:
                   1560: Дарья Русакова
                   1561: 
                   1562: Вопрос 12:
                   1563: В шестидесятых годах в США на НИХ, как ни парадоксально, печатали фразу
                   1564: "не сгибать, не протыкать и не сминать". Назовите ИХ.
                   1565: 
                   1566: Ответ:
                   1567: Перфокарты.
                   1568: 
                   1569: Источник:
                   1570: У. Айзексон. Стив Джобс.
                   1571: https://books.google.ru/books?id=xSWOMjMSzi8C&pg=PT86#v=onepage&q&f=false
                   1572: 
                   1573: Автор:
                   1574: Михаил Скипский
                   1575: 
                   1576: Вопрос 13:
                   1577: Будучи в Ханты-Мансийске, автор вопроса заметил, что у блондинки
1.2     ! rubashki 1578: Вероники Витковой даже ПРОПУСК. Заполните пропуск двумя словами на одну
        !          1579: букву.
1.1       rubashki 1580: 
                   1581: Ответ:
                   1582: Ружье розовое.
                   1583: 
                   1584: Источник:
                   1585: Этап Кубка мира по биатлону в Ханты-Мансийске 2014/15 гг.
                   1586: 
                   1587: Автор:
                   1588: Алексей Шпильман
                   1589: 
                   1590: Вопрос 14:
                   1591: За тридцать лет своей театральной деятельности ОН сыграл много
                   1592: запоминающихся ролей: фурию, развратника, Юпитера и другие. Назовите
                   1593: роль, которая стала ЕГО кульминацией.
                   1594: 
                   1595: Ответ:
                   1596: Солнце.
                   1597: 
                   1598: Комментарий:
                   1599: Людовик XIV даже получил от этой роли прозвище "Солнце". Подсказками
                   1600: служат номер вопроса и слово "кульминация", одно из значений которого -
                   1601: момент прохождения светила через небесный меридиан в процессе его
                   1602: суточного движения.
                   1603: 
                   1604: Источник:
                   1605: Журнал "Вопросы истории", 2014, N 8. - С. 126-127.
                   1606: 
                   1607: Автор:
                   1608: Сергей Виватенко
                   1609: 
                   1610: Вопрос 15:
                   1611: Шведский детский писатель Ульф Старк полушутливо отмечает, что родители,
                   1612: пытаясь пробудить у ребенка интерес к чтению с самого раннего детства,
                   1613: невольно достигают противоположного результата. Поэтому тем родителям,
                   1614: которые хотят, чтобы их дети воспринимали книги с действительно живым
                   1615: интересом, он рекомендует не ДЕЛАТЬ ЭТОГО. Не делать чего?
                   1616: 
                   1617: Ответ:
                   1618: Не читать детям на ночь.
                   1619: 
                   1620: Комментарий:
                   1621: Неудивительно, что книги нагоняют на многих детей сон: это плоды
                   1622: многолетней привычки, говорит Ульф Старк.
                   1623: 
                   1624: Источник:
                   1625: Лекция Ульфа Старка на Днях шведского языка в Москве, 30.09.2015 г.
                   1626: 
                   1627: Автор:
                   1628: Жанна Подоляк, Дарья Русакова
                   1629: 
                   1630: Тур:
6 тур. "Eclipse"
                   1631: 
                   1632: Дата:
                   1633: 11-Oct-2015
                   1634: 
                   1635: Редактор:
                   1636: Наталия Новыш, Владислав Король, Антон Бочкарёв
                   1637: 
                   1638: Инфо:
                   1639: Редакторы благодарят за помощь в подготовке пакета игроков команды
                   1640: "Eclipse" Ивана Беляева, Александра Людикайнена, Анну Резникову и Ивана
                   1641: Старикова, а также Екатерину Андрющенко (Рига), Руслана Батдалова
                   1642: (Рига), Бориса Гуревича (Саратов), Елизавету Иванову (Саратов),
                   1643: Екатерину Свешникову (Саратов), Ивана Суманеева (Сыктывкар -
                   1644: Санкт-Петербург), Игоря Философова (Саратов).
                   1645: 
                   1646: Вопрос 1:
                   1647:    <раздатка>
                   1648:    Я даже не успел испугаться, когда свободной рукой Иона швырнул меня,
                   1649: как мячик, в снег, а откуда-то сбоку раздался выстрел. И уже из сугроба
                   1650: я увидел, как одним прыжком Иона оказался на моем прежнем месте,
                   1651: прикрывая меня от невидимого стрелка.
                   1652:    </раздатка>
                   1653:    Евгений Водолазкин пишет, что в пределах текста ОНО приближает героя
                   1654: к бессмертию. Назовите ЕГО.
                   1655: 
                   1656: Ответ:
                   1657: Повествование от первого лица.
                   1658: 
                   1659: Зачет:
                   1660: По словосочетанию "первое лицо".
                   1661: 
                   1662: Комментарий:
                   1663: Мы раздали цитату из романа Евгения Водолазкина "Похищение Европы", в
                   1664: которой героя-рассказчика в очередной раз спасают. По мнению
                   1665: Водолазкина, "повествование от первого лица тем и хорошо, что лицо это
                   1666: гарантированно выживает... его умение не умирать феноменально".
                   1667: 
                   1668: Источник:
                   1669: Е.Г. Водолазкин. Похищение Европы. http://www.flibusta.net/b/338557/read
                   1670: 
                   1671: Автор:
                   1672: Наталия Новыш (Санкт-Петербург)
                   1673: 
                   1674: Вопрос 2:
                   1675: Внимание, в вопросе есть замена.
                   1676:    Книга англичанина Джеймса Мэя учит вещам, которые должен уметь каждый
                   1677: мужчина. В начале книги подробно рассказывается о том, как забить
                   1678: гвоздь. Какое девятнадцатибуквенное слово заменено в тексте вопроса?
                   1679: 
                   1680: Ответ:
                   1681: Одиннадцатиметровый.
                   1682: 
                   1683: Комментарий:
                   1684: По словам Мэя, "когда надо пробить серию пенальти, английская команда
                   1685: может, не дожидаясь результата, отправляться в душевую". Действительно,
                   1686: с 1990 года сборная Англии вылетала из розыгрышей чемпионатов мира и
                   1687: Европы по пенальти шесть раз (ЧМ 1990, 1998, 2008, ЧЕ 1996, 2004, 2012),
                   1688: выиграв всего одну серию из семи. О том, как забить гвоздь, в книге тоже
                   1689: рассказывается, но пенальти, видимо, важнее.
                   1690: 
                   1691: Источник:
                   1692:    1. Дж. Мэй, У. Маклин. Мужская лаборатория Джеймса Мэя. Книга о
                   1693: полезных вещах. http://www.flibusta.net/b/375053/read
                   1694:    2. http://www.sovsport.ru/gazeta/article-item/726883
                   1695: 
                   1696: Автор:
                   1697: Владислав Король (Москва)
                   1698: 
                   1699: Вопрос 3:
                   1700: По одной версии, ОНИ были плодом воображения сумасшедшей бездомной,
                   1701: ночевавшей в парке на скамейке рядом с фонтаном. Назовите ИХ одним
                   1702: словом.
                   1703: 
                   1704: Ответ:
                   1705: Друзья.
                   1706: 
                   1707: Зачет:
                   1708: Friends.
                   1709: 
                   1710: Комментарий:
                   1711: Возможно, все события сериала "Друзья" были безумной фантазией самой
                   1712: странной его героини - Фиби Буффе, сочинившей себе воображаемых друзей,
                   1713: и на самом деле она много лет наблюдала за жизнью незнакомых людей через
                   1714: окно "Центральной кофейни". Кстати, в самом сериале говорится, что Фиби
                   1715: в юности некоторое время бродяжничала. Скамейка и фонтан присутствуют в
                   1716: заставке всех сезонов сериала.
                   1717: 
                   1718: Источник:
                   1719:    1. http://www.ivi.ru/titr/motor/conspiracy?s=tw
                   1720:    2. http://ru.wikipedia.org/wiki/Фиби_Буффе
                   1721: 
                   1722: Автор:
                   1723: Наталия Новыш (Санкт-Петербург)
                   1724: 
                   1725: Вопрос 4:
                   1726: Внимание, в вопросе есть замена.
                   1727:    В молодости герой Дмитрия Горчева играл в вокально-инструментальном
                   1728: ансамбле под названием "Стремление". У музыкантов было три гитары и
                   1729: ударная установка, а вот АЛЬФА ансамблю не полагалась по смете. Напишите
                   1730: название, замененное АЛЬФОЙ.
                   1731: 
                   1732: Ответ:
                   1733: "Ионика".
                   1734: 
                   1735: Комментарий:
                   1736: Вот такая вот музыка. Такая, блин, вечная молодость.
                   1737: 
                   1738: Источник:
                   1739:    1. Д.А. Горчев. Жизнь без Карло. Музыка для экзальтированных старцев.
                   1740: http://www.flibusta.net/b/391382/read
                   1741:    2. Группа "Чиж и Ко", песня "Вечная молодость".
                   1742: http://www.chizh.net/txt/5.htm
                   1743: 
                   1744: Автор:
                   1745: Владислав Король (Москва)
                   1746: 
                   1747: Вопрос 5:
                   1748: Герой Кьеркегора пытается исподволь навести свою невесту на мысль о
                   1749: разрыве помолвки. Так, брошенный им намек на обычай обмениваться
                   1750: кольцами застал ее во время... Ответьте, использовав несклоняемое слово:
                   1751: во время чего?
                   1752: 
                   1753: Ответ:
                   1754: Серсо.
                   1755: 
                   1756: Комментарий:
                   1757: "Брошенный" - тоже намек. Серсо - игра в обруч, который особой палочкой
                   1758: подкидывается в воздух и затем ловится на ту же палочку (или другим
                   1759: играющим на свою палочку). Героиня бросает два кольца так, что они
                   1760: разлетаются в разные стороны и их нельзя поймать.
                   1761: 
                   1762: Источник:
                   1763: С. Кьеркегор. Дневник обольстителя.
                   1764: http://www.flibusta.net/b/141539/read
                   1765: 
                   1766: Автор:
                   1767: Наталия Новыш (Санкт-Петербург)
                   1768: 
                   1769: Вопрос 6:
                   1770: [Ведущему: читать медленно, под запись.]
                   1771:    амуры - так, подчеркивая преданность Константину Аркадьевичу,
                   1772: называют себя студенты и выпускники его курсов. Напишите шесть букв,
                   1773: пропущенных в вопросе.
                   1774: 
                   1775: Ответ:
                   1776: С, а, й, к, и, н.
                   1777: 
                   1778: Комментарий:
                   1779: Неологизм образован от фамилии Райкин и слова "самурай". Сайт,
                   1780: посвященный выпускникам, так и назван - samuraykiny.ru.
                   1781: 
                   1782: Источник:
                   1783:    1. Программа "Главная роль" на телеканале "Культура", эфир от
                   1784: 06.10.2014 г.
                   1785:    2. http://www.samuraykiny.ru/
                   1786: 
                   1787: Автор:
                   1788: Наталия Новыш (Санкт-Петербург)
                   1789: 
                   1790: Вопрос 7:
                   1791: Внимание, в вопросе есть замена.
                   1792:    Патриотичный Цвингли писал: "Если человек пришивает белый крест, он
                   1793: провозглашает свое желание быть МЕНЬШЕВИКОМ". В романе первой половины
                   1794: XX века слово "МЕНЬШЕВИК" и производные от него встречаются около ста
                   1795: раз. Назовите этот роман.
                   1796: 
                   1797: Ответ:
                   1798: "Унесенные ветром".
                   1799: 
                   1800: Комментарий:
                   1801: Ульрих Цвингли был патриотом Швейцарской Конфедерации, белый крест -
                   1802: один из национальных символов Швейцарии. В романе, действие которого
                   1803: разворачивается во время Гражданской войны в США, конфедераты
                   1804: упоминаются постоянно. В состав Конфедерации вошла меньшая часть
                   1805: довоенных штатов США.
                   1806: 
                   1807: Источник:
                   1808:    1. А. Макграт. Введение в христианское богословие.
                   1809: http://www.flibusta.net/b/317773/read
                   1810:    2. М. Митчелл. Унесенные ветром.
                   1811: http://www.flibusta.net/b/340122/read
                   1812:    3. http://ru.wikipedia.org/wiki/Гражданская_война_в_США
                   1813: 
                   1814: Автор:
                   1815: Антон Бочкарёв (Санкт-Петербург)
                   1816: 
                   1817: Вопрос 8:
                   1818: Концерты скандального музыканта Джи Джи Аллина часто заканчивались
                   1819: прибытием полиции. Можно сказать, что Аллин буквально следовал принципу,
                   1820: известному еще до нашей эры. Сформулируйте этот принцип.
                   1821: 
                   1822: Ответ:
                   1823: Всё свое ношу с собой.
                   1824: 
                   1825: Зачет:
                   1826: Omnia mea mecum porto.
                   1827: 
                   1828: Комментарий:
                   1829: Первоначальный смысл выражения заключался в том, что настоящее богатство
                   1830: человека - это духовное богатство, неосязаемые знания. Аллину же часто
                   1831: приходилось в спешке покидать места выступлений и переезжать в другие
                   1832: города. Поскольку личного имущества у него было очень мало, ничто на
                   1833: месте его не держало и делать это было гораздо проще.
                   1834: 
                   1835: Источник:
                   1836:    1. Фильм "Ненавистный", 36-я минута.
                   1837:    2. http://ru.wikipedia.org/wiki/Omnia_mea_mecum_porto
                   1838: 
                   1839: Автор:
                   1840: Владислав Король (Москва)
                   1841: 
                   1842: Вопрос 9:
                   1843: Герой Елены Катишонок послушно выполняет все указания жены, но всё равно
                   1844: вынужден периодически выслушивать ее назидательные монологи. В этих
                   1845: монологах жена часто употребляет одно слово, которое выговаривает
                   1846: особенно четко, и в такие моменты муж представляет себе готовый к
                   1847: стрельбе полк. Напишите это слово.
                   1848: 
                   1849: Ответ:
                   1850: Дисциплина.
                   1851: 
                   1852: Комментарий:
                   1853: Полк, готовый к стрельбе по команде "Пли!". Жена любила отчетливо, по
                   1854: слогам произносить "Дис-ци-пли-на".
                   1855: 
                   1856: Источник:
                   1857: Е.А. Катишонок. Жили-были старик со старухой.
                   1858: http://www.flibusta.net/b/236724/read
                   1859: 
                   1860: Автор:
                   1861: Наталия Новыш (Санкт-Петербург)
                   1862: 
                   1863: Вопрос 10:
                   1864: По одной из версий, известный оборот появился из-за того, что восемь
                   1865: человек воспринимались бы как большинство. Впрочем, по другой версии,
                   1866: причина в том, что ОН был только кандидатом. Назовите ЕГО.
                   1867: 
                   1868: Ответ:
                   1869: Шепилов.
                   1870: 
                   1871: Комментарий:
                   1872: "Ожесточенное сопротивление пыталась оказать осуществлению ленинского
                   1873: курса, намеченного XX съездом партии, фракционная антипартийная группа,
                   1874: в которую входили Молотов, Каганович, Маленков, Ворошилов, Булганин,
                   1875: Первухин, Сабуров и примкнувший к ним Шепилов". В отличие от других
                   1876: участников "антипартийной группы", Шепилов был лишь кандидатом в
                   1877: президиум ЦК КПСС.
                   1878: 
                   1879: Источник:
                   1880: http://ru.wikipedia.org/wiki/Шепилов,_Дмитрий_Трофимович
                   1881: 
                   1882: Автор:
                   1883: Антон Бочкарёв (Санкт-Петербург)
                   1884: 
                   1885: Вопрос 11:
                   1886: В стихотворении Агостиньо Нето ОНА сравнивается с вопросительным знаком,
                   1887: а ОН - с точкой. Третий ОН не стал точкой, и в 2018 году должен
                   1888: появиться еще один. Назовите ЕГО и ЕЕ.
                   1889: 
                   1890: Ответ:
                   1891: Мадагаскар, Африка.
                   1892: 
                   1893: Комментарий:
                   1894: Четвертый мультфильм из серии "Мадагаскар". Агостиньо Нето - африканский
                   1895: поэт и первый президент Анголы.
                   1896: 
                   1897: Источник:
                   1898:    1. http://geo.1september.ru/article.php?ID=200100208
                   1899:    2. http://en.wikipedia.org/wiki/Madagascar_(franchise)#Madagascar_4_.28TBA.29
                   1900: 
                   1901: Автор:
                   1902: Иван Стариков (Санкт-Петербург)
                   1903: 
                   1904: Вопрос 12:
                   1905: ЭТОТ ЧЕЛОВЕК рассказывал, как после революции Андрей Платонов ходил по
                   1906: деревням и слушал разговоры о том, что смерти больше не будет, и только
                   1907: когда один дедушка умер, все поняли, что что-то не так. Назовите ЭТОГО
                   1908: ЧЕЛОВЕКА.
                   1909: 
                   1910: Ответ:
                   1911: Егор Летов.
                   1912: 
                   1913: Комментарий:
                   1914: В известной песне Егора Летова "Всё идет по плану" содержится
                   1915: предположение о том, что при коммунизме "наверное, вообще не надо будет
                   1916: умирать". Кстати, покойный дедушка в этой песне тоже упоминается.
                   1917: 
                   1918: Источник:
                   1919:    1. http://www.gr-oborona.ru/pub/anarhi/1056979282.html
                   1920:    2. http://www.gr-oborona.ru/texts/1056899068.html
                   1921: 
                   1922: Автор:
                   1923: Антон Бочкарёв (Санкт-Петербург)
                   1924: 
                   1925: Вопрос 13:
                   1926: Говоря о "НЕЙ", литературовед Евгений Жаринов упомянул щепки от "корабля
                   1927: эпохи Возрождения". Назовите "ЕЕ" одним словом.
                   1928: 
                   1929: Ответ:
                   1930: "Буря".
                   1931: 
                   1932: Комментарий:
                   1933: А вы думали, вопросов о Шекспире не будет? По мнению Жаринова, в "Буре"
                   1934: отражается крушение мировоззрения, типичного для эпохи Возрождения, и
                   1935: уже заметны элементы барокко.
                   1936: 
                   1937: Источник:
                   1938: http://www.muzcentrum.ru/orpheusradio/programs/labyrinthsecrethistorymusic/13642
                   1939: 
                   1940: Автор:
                   1941: Владислав Король (Москва)
                   1942: 
                   1943: Вопрос 14:
                   1944: Недалеко от подмосковного наукограда Пущино находится объект, несколько
                   1945: столетий назад имевший важное стратегическое значение. Одно из шуточных
                   1946: прозвищ Пущина делает его тезкой зарубежного города. Какого?
                   1947: 
                   1948: Ответ:
                   1949: Оксфорд.
                   1950: 
                   1951: Комментарий:
                   1952: Недалеко от Пущино находится брод через Оку, который неоднократно
                   1953: использовался для переправы московских и татарских войск. Название
                   1954: университетского Оксфорда переводится с английского как "бычий брод".
                   1955: 
                   1956: Источник:
                   1957: Рассказ экскурсовода на экскурсии по Пущино, лето 2013 г.
                   1958: 
                   1959: Автор:
                   1960: Владислав Король (Москва)
                   1961: 
                   1962: Вопрос 15:
                   1963: Внимание, в вопросе есть замена.
                   1964:    В стихотворении Андрея Машнина чайник СДЕЛАЛ ЭТО и свистит. Согласно
                   1965: Фразеологическому словарю русского языка, СДЕЛАТЬ ЭТО - "испытать
                   1966: что-либо до конца, до предела". Какие слова мы заменили словами "СДЕЛАТЬ
                   1967: ЭТО"?
                   1968: 
                   1969: Ответ:
                   1970: Дойти до точки.
                   1971: 
                   1972: Комментарий:
                   1973: Чайник дошел до точки кипения. Лирический герой Андрея Машнина раздражен
                   1974: и недоволен жизнью, и чайник ему под стать.
                   1975: 
                   1976: Источник:
                   1977:    1. http://www.stihi.ru/2000/12/15-202/
                   1978:    2. http://phraseology.academic.ru/3670/
                   1979: 
                   1980: Автор:
                   1981: Антон Бочкарёв (Санкт-Петербург)
                   1982: 
                   1983: Тур:
7 тур. "Мискузи"
                   1984: 
                   1985: Дата:
                   1986: 01-Nov-2015
                   1987: 
                   1988: Вопрос 1:
                   1989: Актер Майкл Кейн вспоминает, что уже в молодости был очень амбициозен и
                   1990: отказывался от сценариев, на которых были ОНИ. Африканский художник Дрис
                   1991: Мааруфи день и ночь создает картины с ИХ помощью. Назовите ИХ.
                   1992: 
                   1993: Ответ:
                   1994: Пятна от кофейных чашек.
                   1995: 
                   1996: Зачет:
                   1997: Пятна кофе, пятна от кофе и т.п. по смыслу. Незачет: Пятна от чая.
                   1998: 
                   1999: Комментарий:
                   2000: Актер Майкл Кейн считал сценарии, которые уже кому-то предлагались,
                   2001: недостойными себя. Африканский художник создает картины с помощью пятен
                   2002: от чашек с любимым напитком.
                   2003: 
                   2004: Источник:
                   2005:    1. http://ria.ru/weekend_cinema/20130314/808111640.html
                   2006:    2. http://www.art-eda.info/po-sledam-kofejnyx-pyaten.html
                   2007: 
                   2008: Автор:
                   2009: Кирилл Ильин
                   2010: 
                   2011: Вопрос 2:
                   2012: Внимание, сейчас ласточки принесут вам раздаточный материал!
                   2013:    (pic: 20150709.jpg)
                   2014:    В состав блюда "Обезьяний ИКС" входят, в частности, мука, какао и
                   2015: большое количество сахара. Какие два слова мы заменили на ИКС?
                   2016: 
                   2017: Ответ:
                   2018: Сладкий хлебушек.
                   2019: 
                   2020: Зачет:
                   2021: Сладкий хлеб.
                   2022: 
                   2023: Комментарий:
                   2024: Сахар придает сладкий вкус, а какао - характерный цвет этому хлебушку.
                   2025: Да, и недаром раздаточный материал именно "вам принесли". Обыгрывается
                   2026: аллюзия на фильм "Зеленый слоник".
                   2027: 
                   2028: Источник:
                   2029: http://www.povarenok.ru/recipes/show/57283/
                   2030: 
                   2031: Автор:
                   2032: Александр Евсюков
                   2033: 
                   2034: Вопрос 3:
                   2035: Внимание, в вопросе есть замена.
                   2036:    В одной компьютерной игре для восстановления жизней герою необходимо
                   2037: подбирать сердца. Как правило, они розовые, но на последнем уровне
                   2038: встречаются и красные, поскольку там есть ПРОПУСК. По одной из версий,
                   2039: ПРОПУСК на гербе Азербайджанской ССР символизировал Солнце, восходящее
                   2040: из моря. Заполните пропуск двумя словами.
                   2041: 
                   2042: Ответ:
                   2043: Розовый фон.
                   2044: 
                   2045: Комментарий:
                   2046: Некоторые сердца красные, поскольку на розовом фоне их было бы просто не
                   2047: видно.
                   2048: 
                   2049: Источник:
                   2050:    1. Игра "Papa Louie 3: When Sundaes Attack!".
                   2051:    2. https://sites.google.com/site/svsvet/simvolika/azerbajdzan
                   2052:    3. http://cccp-15.narod.ru/gerb_az.htm
                   2053: 
                   2054: Автор:
                   2055: Виктор Дзекановский, Александр Евсюков
                   2056: 
                   2057: Вопрос 4:
                   2058: Внимание, тетраблиц! Все четыре вопроса задаются сразу. Ответ сдается по
                   2059: истечении минуты на одной карточке.
                   2060:    1. В русском языке ИХ название происходит от голландского слова,
                   2061: означающего "глазница". Назовите ИХ одним словом.
                   2062:    2. ОНИ - это еще и разновидность фольклорных элементов - повторение в
                   2063: начале следующей строки конца предыдущей. Назовите ИХ одним словом.
                   2064:    3. ОНИ бывают, например, зубчатыми, фрикционными и карданными.
                   2065: Назовите ИХ одним словом.
                   2066:    4. Человек, который считается одним из лучших в истории специалистов
                   2067: по НИМ, в 2000 году снялся в фильме под названием "Затяжной прыжок".
                   2068: Назовите ИХ одним словом.
                   2069:    Внимание! Мы не просим вас отвечать на все эти вопросы. Ответьте, что
                   2070: мы заменили в преамбуле этого блица.
                   2071: 
                   2072: Ответ:
                   2073: Квадрупл.
                   2074: 
                   2075: Зачет:
                   2076: Квадрупл-дабл.
                   2077: 
                   2078: Комментарий:
                   2079: Ответы на эти вопросы - очки, перехваты, передачи и подборы.
                   2080: Квадрупл-дабл - баскетбольный термин, означающий набор игроком в течение
                   2081: одного матча двузначного количества пунктов одновременно в четырех из
                   2082: пяти основных статистических показателей (очки, подборы, передачи,
                   2083: перехваты, блокшоты). Это был не тетраблиц, а квадрупл-блиц.
                   2084: 
                   2085: Источник:
                   2086:    1. http://ru.wikipedia.org/wiki/Очки
                   2087:    2. http://vsesochineniya.ru/sochinenie-na-temu-folklornye-elementy-v-poeme-m-yu-lermnotova-pesnya-pro-kup%C2%ADca-kalashnikova.html
                   2088:    3. http://ru.wikipedia.org/wiki/Механическая_передача
                   2089:    4. http://ru.wikipedia.org/wiki/Родман,_Деннис
                   2090: 
                   2091: Автор:
                   2092: Кирилл Ильин
                   2093: 
                   2094: Вопрос 5:
                   2095: В фильме "Разборки в стиле кунг-фу" героиня, говоря о молодом мастере
                   2096: кунг-фу, который в одиночку сражается сразу с несколькими десятками
                   2097: вооруженных противников, замечает, что тот мог бы стать отличным ИМ.
                   2098: Назовите ЕГО словом французского происхождения.
                   2099: 
                   2100: Ответ:
                   2101: Каскадер.
                   2102: 
                   2103: Источник:
                   2104: Фильм "Разборки в стиле кунг-фу" (2004), реж. Стивен Чоу.
                   2105: 
                   2106: Автор:
                   2107: Виктор Дзекановский
                   2108: 
                   2109: Вопрос 6:
                   2110: (pic: 20150710.jpg)
                   2111:    Как ни странно, перед вами фрагмент из АЛЬФЫ под названием "Зеркала",
                   2112: созданной в 1966 году. Назовите АЛЬФУ.
                   2113: 
                   2114: Ответ:
                   2115: Поэма.
                   2116: 
                   2117: Комментарий:
                   2118: Это фрагмент из поэмы Семена Кирсанова "Зеркала", написанной в 1966
                   2119: году:
                   2120:    Возможно, что луч, ложась на стекло под углом,
                   2121:    Придает составным особый уклон,
                   2122:    И частицы встают, как иглы ежа: каждая - снимок,
                   2123:    Колючий начёс световых невидимок.
                   2124:    Верно ли? Спорно ли?
                   2125:    Просто, как в формуле:
                   2126:    Эн квадрат равняется единице плюс дробь, где числитель четыре пи эн е
                   2127: квадрат,
                   2128:    А знаменатель некое К?
                   2129: 
                   2130: Источник:
                   2131: http://rupoem.ru/kirsanov/zerkala-na-stene.aspx
                   2132: 
                   2133: Автор:
                   2134: Кирилл Ильин
                   2135: 
                   2136: Вопрос 7:
                   2137: "ОНА" - это название книги народного учителя СССР Виктора Шаталова, в
                   2138: которой рассказывается о школьных буднях. Известное произведение коллеги
                   2139: Шаталова - тоже по сути ОНА. Назовите ЕЕ двумя словами, начинающимися на
                   2140: одну и ту же букву.
                   2141: 
                   2142: Ответ:
                   2143: Педагогическая проза. Незачет: Педагогическая повесть; педагогическая
                   2144: поэма.
                   2145: 
                   2146: Комментарий:
                   2147: В.Ф. Шаталов в своей книге рассказывает о прозе школьной жизни;
                   2148: "Педагогическая поэма" А.С. Макаренко, несмотря на название, является
                   2149: прозаическим произведением.
                   2150: 
                   2151: Источник:
                   2152:    1. http://www.nsportal.ru/gp/2012/12/shatalov-vf-pedagogicheskaya-proza
                   2153:    2. http://ru.wikipedia.org/wiki/Педагогическая_поэма
                   2154: 
                   2155: Автор:
                   2156: Александр Евсюков
                   2157: 
                   2158: Вопрос 8:
                   2159: 23 сентября 2015 года Хидекичи Миязаки пробежал 100 метров за 42,22
                   2160: секунды и был объявлен мировым рекордсменом. Однако вскоре стало
                   2161: известно, что 28 июня Станислав Ковальски пробежал ту же дистанцию за
                   2162: 34,50 секунды, и его результат превышает рекорд Миязаки не только на
                   2163: семь с лишним секунд, но и на семьдесят два... Чего?
                   2164: 
                   2165: Ответ:
                   2166: Дня.
                   2167: 
                   2168: Комментарий:
                   2169: Они являются самыми возрастными бегунами. Миязаки в день рекордного
                   2170: забега исполнилось 105 лет и 1 день от роду, а Ковальски - 105 лет и 73
                   2171: дня.
                   2172: 
                   2173: Источник:
                   2174: http://skirun.ru/2015/09/27/hidekichi-miyazaki-false-guinness-world-record-holder/
                   2175: 
                   2176: Автор:
                   2177: Кирилл Ильин
                   2178: 
                   2179: Вопрос 9:
                   2180: Рассказывая об указании Владимира Ленина о выдворении за пределы СССР
                   2181: некоторых деятелей культуры, "Большая энциклопедия Кирилла и Мефодия"
                   2182: приводит слова Пастернака: "Он управлял теченьем мысли, и только потому
1.2     ! rubashki 2183: - страной". Точнее, поясняют авторы, мыслью ПРОПУСК, но не философа.
1.1       rubashki 2184: Заполните пропуск.
                   2185: 
                   2186: Ответ:
                   2187: Кухарки.
                   2188: 
                   2189: Комментарий:
                   2190: Достаточно известна фраза Ленина: "Любая кухарка должна уметь управлять
                   2191: государством".
                   2192: 
                   2193: Источник:
                   2194: БЭКМ-2009, статья "Ленин, Владимир Ильич".
                   2195: 
                   2196: Автор:
                   2197: Александр Евсюков
                   2198: 
                   2199: Вопрос 10:
                   2200: (pic: 20150711.jpg)
                   2201:    Переведите розданное вам слово с датского языка на русский.
                   2202: 
                   2203: Ответ:
                   2204: Остров.
                   2205: 
                   2206: Комментарий:
                   2207: В состав Дании входит Гренландия - самый большой остров на Земле.
                   2208: 
                   2209: Источник:
                   2210: https://translate.google.ru/#da/ru/%C3%B8
                   2211: 
                   2212: Автор:
                   2213: Петр Назаров
                   2214: 
                   2215: Вопрос 11:
                   2216: В одной из сцен фильма "Возвращение помидоров-убийц" героям нужны
                   2217: деньги. В следующей сцене один из героев поочередно держит в руке банку
                   2218: "Пепси-Колы" и шоколадный батончик. На что нужны были деньги в первой из
                   2219: этих сцен?
                   2220: 
                   2221: Ответ:
                   2222: На продолжение съемок фильма.
                   2223: 
                   2224: Зачет:
                   2225: По смыслу.
                   2226: 
                   2227: Комментарий:
                   2228: Поскольку денег на продолжение съемок фильма не было, пришлось добавить
                   2229: в картину продакт-плейсмента.
                   2230: 
                   2231: Источник:
                   2232: Фильм "Возвращение помидоров-убийц" (1988), реж. Джон Де Белло.
                   2233: 
                   2234: Автор:
                   2235: Виктор Дзекановский
                   2236: 
                   2237: Вопрос 12:
                   2238: Эпизод мультсериала "Южный парк", в котором описывается возможность
                   2239: катастрофы из-за глобального потепления, называется "За два дня до...".
                   2240: До чего?
                   2241: 
                   2242: Ответ:
                   2243: Послезавтра.
                   2244: 
                   2245: Комментарий:
                   2246: Аллюзия на знаменитый фильм Роланда Эммериха. Через два дня и в самом
                   2247: деле наступает послезавтра. За два дня до послезавтра - это как раз
                   2248: сегодня.
                   2249: 
                   2250: Источник:
                   2251: http://ru.wikipedia.org/wiki/За_два_дня_до_послезавтра
                   2252: 
                   2253: Автор:
                   2254: Виктор Дзекановский
                   2255: 
                   2256: Вопрос 13:
                   2257: Австралийская супермодель во время интервью рассказала, что ее рацион на
                   2258: 80% состоит из здоровой пищи и на 20% - из не очень здоровой.
                   2259: Немногословный автор вопроса назвал этот принцип своеобразным ИМ.
                   2260: Назовите ЕГО двумя словами, которые начинаются на разные буквы.
                   2261: 
                   2262: Ответ:
                   2263: Правило Миранды.
                   2264: 
                   2265: Комментарий:
                   2266: Это была супермодель Миранда Керр. А принцип Парето здесь вообще ни при
                   2267: чем.
                   2268: 
                   2269: Источник:
                   2270: http://www.youtube.com/watch?v=tLgWenn1IvI
                   2271: 
                   2272: Автор:
                   2273: Кирилл Ильин
                   2274: 
                   2275: Вопрос 14:
                   2276: ОН скончался прямо с приказом в руках в августе 1983 года во время
                   2277: очередной поездки по местам боевой славы. Назовите ЕГО.
                   2278: 
                   2279: Ответ:
                   2280: [Юрий Борисович] Левитан.
                   2281: 
                   2282: Комментарий:
                   2283: Диктор совершал поездку по местам Курской битвы (по случаю 40-летней
                   2284: годовщины) и скончался от сердечного приступа прямо во время чтения
                   2285: приказа.
                   2286: 
                   2287: Источник:
                   2288: Фильм "Голос эпохи" из цикла "Тайны забытых побед".
                   2289: 
                   2290: Автор:
                   2291: Александр Евсюков
                   2292: 
                   2293: Вопрос 15:
                   2294: Надеемся, что вам понравился наш тур.
                   2295:    Описывая произношение арабского звука, обозначаемого буквой "айн",
                   2296: Джей Смарт отмечает, что мышцу, находящуюся возле адамова яблока,
                   2297: европейцы, в отличие от арабов, задействуют только при НЕМ. Назовите ЕГО
                   2298: двумя словами, начинающимися на одну и ту же букву.
                   2299: 
                   2300: Ответ:
                   2301: Рвотный рефлекс.
                   2302: 
                   2303: Комментарий:
                   2304: Гортанный звук "айн" не имеет аналогов в европейских языках, а попытка
                   2305: произнести его может закончиться для европейца, начинающего изучение
                   2306: арабского языка, тошнотой в прямом смысле этого слова. Мы, в свою
                   2307: очередь, надеемся, что у вас останется положительное впечатление о нашем
                   2308: туре и благодарим все команды за участие!
                   2309: 
                   2310: Источник:
                   2311: J.R. Smart. Teach yourself Arabic.
                   2312: 
                   2313: Автор:
                   2314: Михаил Питателев
                   2315: 
                   2316: Тур:
8 тур. "Полосатая неясыть"
                   2317: 
                   2318: Дата:
                   2319: 01-Nov-2015
                   2320: 
                   2321: Инфо:
                   2322: Команда благодарит за помощь в подготовке пакета Максима Поташева
                   2323: (Москва), Евгения Поникарова (Санкт-Петербург), Александра Коробейникова
                   2324: (Саратов - Санкт-Петербург - Москва), Анну Овчинникову и Игоря Колмакова
                   2325: (Тель-Авив), Эдуарда и Ксению Шагал (Хельсинки), Татьяну Харитонову и
                   2326: Александру Башину (Москва), а также координатора XVII чемпионата
                   2327: Санкт-Петербурга Владимира Белкина (Тель-Авив).
                   2328: 
                   2329: Вопрос 1:
                   2330: В 1817 году учащиеся Морского кадетского корпуса совершили учебное
                   2331: плавание из Петербурга в Данию. В пути им велено было вести дневники.
                   2332: Дневники, написанные двумя самыми толковыми кадетами, были впоследствии
                   2333: изданы. Один дневник вел Павел Нахимов. А кто вел другой?
                   2334: 
                   2335: Ответ:
                   2336: [Владимир] Даль.
                   2337: 
                   2338: Комментарий:
                   2339: Даль, как известно, начинал морским офицером. Слова "Дания" и особенно
                   2340: "толковыми" - подсказки. Отцом Владимира Ивановича Даля был обрусевший
                   2341: датчанин Йохан (Иоганн) Кристиан вон Даль.
                   2342: 
                   2343: Источник:
                   2344: "Санкт-Петербургские ведомости", 07.08.2014 г. - С. 1.
                   2345: 
                   2346: Автор:
                   2347: Александр Лихтшангоф
                   2348: 
                   2349: Вопрос 2:
                   2350: Из слов мультипликационного персонажа следует, что ОНА весит не более
                   2351: пяти килограммов. На съемках советского фильма ОНА, вероятно, была из
                   2352: алюминия. Назовите ЕЕ.
                   2353: 
                   2354: Ответ:
                   2355: Кочерга.
                   2356: 
                   2357: Комментарий:
                   2358: Почтальон Печкин классифицирует кочергу как бандероль, а она в СССР
                   2359: должна была весить не более пяти килограммов. В первой части
                   2360: ("Знакомство") фильма "Шерлок Холмс и доктор Ватсон", снятой по мотивам
                   2361: рассказа "Пестрая лента", доктор Гримсби Ройлотт сгибает кочергу,
                   2362: которую затем разгибает Шерлок Холмс.
                   2363: 
                   2364: Источник:
                   2365:    1. http://ru.wikipedia.org/wiki/Бандероль
                   2366:    2. http://www.youtube.com/watch?v=Rw1QIF9B-q8
                   2367:    3. http://wap.221b.borda.ru/?1-9-80-00000065-000-10001-0
                   2368: 
                   2369: Автор:
                   2370: Борис Моносов
                   2371: 
                   2372: Вопрос 3:
                   2373: Южный федеральный университет израсходовал более трех миллиардов рублей
                   2374: на модернизацию образовательного процесса. Мы не спрашиваем, сколько
                   2375: точно они потратили и как поделили. Назовите неологизм, которым назвал
                   2376: эту сумму остроумный журналист "Русского репортера".
                   2377: 
                   2378: Ответ:
                   2379: Пиллиард.
                   2380: 
                   2381: Комментарий:
                   2382: Было потрачено 3,14 млрд. руб., а как "распилили" - неизвестно.
                   2383: 
                   2384: Источник:
                   2385: http://rusrep.ru/article/2012/07/17/univer/
                   2386: 
                   2387: Автор:
                   2388: Александр Лихтшангоф
                   2389: 
                   2390: Вопрос 4:
                   2391: Закончите двумя словами хайку Виктора КагАна: "Пискнул мышонок // от
                   2392: радости свистнул рак // ...".
                   2393: 
                   2394: Ответ:
                   2395: Гора родила.
                   2396: 
                   2397: Зачет:
                   2398: Родила гора.
                   2399: 
                   2400: Комментарий:
                   2401: Контаминация двух русских фразеологизмов: "Гора родила мышь" и "Рак на
                   2402: горе свистнул".
                   2403: 
                   2404: Источник:
                   2405: http://45parallel.net/viktor_kagan/khayku/
                   2406: 
                   2407: Автор:
                   2408: Александр Лихтшангоф
                   2409: 
                   2410: Вопрос 5:
                   2411: В 2005 году газета "Спорт-Экспресс" писала, что если судить матч
                   2412: чемпионата России по футболу приглашают иностранного судью, то этот
                   2413: арбитр похож на НЕЕ. Назовите ЕЕ мужа.
                   2414: 
                   2415: Ответ:
                   2416: [Гай Юлий] Цезарь.
                   2417: 
                   2418: Комментарий:
                   2419: Знаменитая фраза Юлия Цезаря "Жена Цезаря должна быть вне подозрений"
                   2420: часто цитируется в сокращенном и искаженном виде: "Жена Цезаря вне
                   2421: подозрений". Иностранных судей, в отличие от российских, действительно,
                   2422: как правило, не подозревают в предвзятом судействе. Слово "арбитр" -
                   2423: небольшая наводка на античность.
                   2424: 
                   2425: Источник:
                   2426: http://www.sport-express.ru/newspaper/2005-10-17/3_4/
                   2427: 
                   2428: Автор:
                   2429: Мишель Матвеев
                   2430: 
                   2431: Вопрос 6:
                   2432: В Омске на улице, названной в честь одного военачальника, находится
                   2433: бассейн, названный в честь другого военачальника. Назовите этих
                   2434: военачальников в любом порядке.
                   2435: 
                   2436: Ответ:
                   2437: [Василий Иванович] Чапаев, Ермак [Тимофеевич].
                   2438: 
                   2439: Зачет:
                   2440: В любом порядке.
                   2441: 
                   2442: Комментарий:
                   2443: Оба, по преданию, утонули.
                   2444: 
                   2445: Источник:
                   2446: http://www.probasseyn.ru/article/71/
                   2447: 
                   2448: Автор:
                   2449: Борис Моносов
                   2450: 
                   2451: Вопрос 7:
                   2452: В молодой советской республике почти не было ИХ. Возможно, поэтому песня
                   2453: сороковых годов стала широко известна только в семидесятые, после выхода
                   2454: фильма. Назовите ИХ.
                   2455: 
                   2456: Ответ:
                   2457: Партизаны.
                   2458: 
                   2459: Комментарий:
                   2460: Речь о песне "Смуглянка", прозвучавшей в фильме "В бой идут одни
                   2461: "старики"". Партизанское движение в Молдавии не было таким широким, как
                   2462: могло бы показаться из песни. Возможно, из-за того, что республика вошла
                   2463: в состав СССР лишь за год до Великой Отечественной войны.
                   2464: 
                   2465: Источник:
                   2466:    1. http://www.leonid-bykov.ru/fight/9.htm
                   2467:    2. http://ru.wikipedia.org/wiki/Молдавская_Советская_Социалистическая_Республика
                   2468: 
                   2469: Автор:
                   2470: Мишель Матвеев
                   2471: 
                   2472: Вопрос 8:
                   2473: Медики ДеквЕркер и РикО отметили ЕЕ сосископодобные пальцы и отёк
                   2474: кистевых суставов. Один из родственников ЕЕ создателя, судя по всему,
                   2475: тоже был не идеален и, в частности, склонен к полноте. Назовите этого
                   2476: создателя.
                   2477: 
                   2478: Ответ:
                   2479: [Сандро] Боттичелли.
                   2480: 
                   2481: Комментарий:
                   2482: На картине "Рождение Венеры" Венера - вовсе не идеальная красавица.
                   2483: Прозвище "Боттичелли" ("Бочонок") перешло к художнику от старшего брата
                   2484: Джованни, который был толстяком.
                   2485: 
                   2486: Источник:
                   2487:    1. К.И. Шапиро. Травматология и ортопедия в зеркале искусства. -
                   2488: СПб., 1997. - С. 31.
                   2489:    2. http://ru.wikipedia.org/wiki/Сандро_Боттичелли
                   2490: 
                   2491: Автор:
                   2492: Мишель Матвеев
                   2493: 
                   2494: Вопрос 9:
                   2495: Портал "Хэдхантер" пишет, что если начальник без энтузиазма отвечает на
                   2496: Ваши приветствия, а то и вообще не замечает, возможно, он просто не
                   2497: видит смысла в общении с ТАКИМ сотрудником. Герой песни группы
                   2498: "Несчастный случай" говорит: "Я твой нынешний, это значит - ТАКОЙ".
                   2499: Какие два слова, начинающиеся на одну и ту же букву, мы заменили словом
                   2500: "ТАКОЙ"?
                   2501: 
                   2502: Ответ:
                   2503: Будущий бывший.
                   2504: 
                   2505: Комментарий:
                   2506: Рекрутеры говорят, что вышеописанное - косвенные признаки грядущего
                   2507: увольнения.
                   2508: 
                   2509: Источник:
                   2510:    1. http://rabota.mail.ru/article/16393
                   2511:    2. http://www.ns.ru/news/news_10211.html
                   2512: 
                   2513: Автор:
                   2514: Михаил Локшин, Маргарита Савицкая
                   2515: 
                   2516: Вопрос 10:
                   2517:    <раздатка>
                   2518:    Почиет отблеск славы на Тебе
                   2519:    Господней; Божий Дух в Тебя вселен.
                   2520:    Твоей [...] сотворил Господь
                   2521:    И Небеса Небес, и сонмы Сил
                   2522:    &nbsp;
                   2523:    - Таково, значит, твое представление о справедливости? Я рад, что
                   2524: больше не являюсь твоим [...].
                   2525:    </раздатка>
                   2526:    В каждой цитате пропущено одно и то же слово. Напишите его.
                   2527: 
                   2528: Ответ:
                   2529: Десницей.
                   2530: 
                   2531: Зачет:
                   2532: Десница.
                   2533: 
                   2534: Комментарий:
                   2535: В первом случае слово "десница" женского рода, во втором - мужского. В
                   2536: цикле "Песнь Льда и Пламени" десницей короля называется его первый
                   2537: министр.
                   2538: 
                   2539: Источник:
                   2540:    1. Дж. Мильтон. Потерянный рай. http://www.flibusta.net/b/149916/read
                   2541:    2. Дж. Мартин. Игра Престолов. http://www.flibusta.net/b/237689/read
                   2542: 
                   2543: Автор:
                   2544: Мишель Матвеев
                   2545: 
                   2546: Вопрос 11:
                   2547: Киновед Евгений Громов писал о произведении 1965 года: "Мораль сей басни
                   2548: проста и мудра. У настоящего художника нет и не может быть ИХ. Он, по
                   2549: воле собственного сердца, всегда в работе, в творчестве". Назовите ИХ.
                   2550: 
                   2551: Ответ:
                   2552: Каникулы.
                   2553: 
                   2554: Комментарий:
                   2555: Речь идет о мультфильме "Каникулы Бонифация". На отдыхе Бонифаций каждый
                   2556: день с утра до вечера дает представления, забыв о рыбной ловле и купании
                   2557: в озере.
                   2558: 
                   2559: Источник:
                   2560: http://ru.wikipedia.org/wiki/Каникулы_Бонифация
                   2561: 
                   2562: Автор:
                   2563: Михаил Локшин
                   2564: 
                   2565: Вопрос 12:
                   2566: В одной газетной статье рассказывается о половой жизни игуан. Оранжевые
                   2567: самцы игуан - самые сильные. Они захватывают большую территорию и
                   2568: формируют большой гарем. Желтые самцы - самые мелкие, они могут
                   2569: прокрадываться в гарем оранжевых и оплодотворять их самок. Синие самцы
                   2570: слабее оранжевых, гарем и территория у них меньше, поэтому они способны
                   2571: эффективно охранять их от желтых. Воспроизведите название данной статьи,
                   2572: состоящее из трех слов.
                   2573: 
                   2574: Ответ:
                   2575: Камень, ножницы, бумага.
                   2576: 
                   2577: Комментарий:
                   2578: Оранжевые выигрывают у синих, но проигрывают желтым. Синие выигрывают у
                   2579: желтых, но проигрывают оранжевым. Желтые выигрывают у оранжевых, но
                   2580: проигрывают синим. Ситуация напомнила автору статьи известную игру.
                   2581: 
                   2582: Источник:
                   2583: "Троицкий вариант - Наука", N 18 (162), 09.09.2014 г. - С. 13.
                   2584: 
                   2585: Автор:
                   2586: Александр Лихтшангоф
                   2587: 
                   2588: Вопрос 13:
                   2589: Когда во время Первой мировой войны британские женщины ДЕЛАЛИ ЭТО,
                   2590: причиной, по утверждению Александры Борисенко, был их патриотизм. Шутят,
                   2591: что другая британская женщина, СДЕЛАВШАЯ ЭТО по максимуму, тоже была
                   2592: патриоткой и заботилась о населении страны. Назовите то, что создала эта
                   2593: женщина.
                   2594: 
                   2595: Ответ:
                   2596: Мини-юбка.
                   2597: 
                   2598: Комментарий:
                   2599: ДЕЛАТЬ ЭТО - укорачивать юбку. Более короткие юбки позволяли снизить
                   2600: расход материалов на одежду. Модельер Мэри Куант, Офицер ордена
                   2601: Британской империи, считается создательницей мини-юбки; шутники
                   2602: утверждали, что орден был ей вручен именно за создание мини-юбки и вклад
                   2603: в увеличение рождаемости.
                   2604: 
                   2605: Источник:
                   2606:    1. А. Борисенко. Дела семейные. // Только не дворецкий. Золотой век
                   2607: британского детектива. http://www.flibusta.net/b/373269/read#t88
                   2608:    2. http://ru.wikipedia.org/wiki/Куант,_Мэри
                   2609: 
                   2610: Автор:
                   2611: Кристина Кораблина
                   2612: 
                   2613: Вопрос 14:
                   2614: ПЕРВОЕ альбома "Dark side of the moon" было "SECOND". Недавно "ПЕРВОЕ" и
                   2615: "SECOND" публично продемонстрировали свои творения с разницей в неделю.
                   2616: Что мы заменили словами "ПЕРВОЕ" и "SECOND"?
                   2617: 
                   2618: Ответ:
                   2619: Рабочее название, Eclipse.
                   2620: 
                   2621: Зачет:
                   2622: Рабочее название, Эклипс.
                   2623: 
                   2624: Комментарий:
                   2625: Команда "Рабочее название" проводила третий тур XVII чемпионата
                   2626: Санкт-Петербурга 4 октября, команда "Eclipse" - шестой тур 11 октября.
                   2627: Также в формулировке вопроса присутствует отсылка к вопросам номер 1 и
                   2628: 14 тура команды "Рабочее название".
                   2629: 
                   2630: Источник:
                   2631:    1. http://ru.wikipedia.org/wiki/The_Dark_Side_of_the_Moon
                   2632:    2. http://www.spbchamp.aerobrain.ru/
                   2633: 
                   2634: Автор:
                   2635: Никита Иваньков
                   2636: 
                   2637: Вопрос 15:
                   2638: В романе Акунина француз, проигрывая британцу в шахматы, упоминает ЕЕ,
                   2639: подразумевая, по всей видимости, ТАКУЮ ЕЕ. Кто в 1946 году написал
                   2640: "ЭТАКУЮ ЕЕ"?
                   2641: 
                   2642: Ответ:
                   2643: [Александр Александрович] Фадеев.
                   2644: 
                   2645: Комментарий:
                   2646: Француз повторяет фразу, сказанную, по легенде, при Ватерлоо: "Гвардия
                   2647: умирает, но не сдается". Старая гвардия - французское элитное
                   2648: подразделение Императорской гвардии периода наполеоновских войн. А роман
                   2649: "Молодая гвардия" написал Александр Фадеев.
                   2650: 
                   2651: Источник:
                   2652:    1. Б. Акунин. Турецкий гамбит. http://www.flibusta.net/b/305713/read
                   2653:    2. http://ru.wikipedia.org/wiki/Гвардия_умирает,_но_не_сдаётся
                   2654:    3. http://ru.wikipedia.org/wiki/Старая_гвардия_(Первая_империя)
                   2655:    4. http://ru.wikipedia.org/wiki/Молодая_гвардия_(роман)
                   2656: 
                   2657: Автор:
                   2658: Мишель Матвеев
                   2659: 
                   2660: Тур:
9 тур. "Ноев камбэк"
                   2661: 
                   2662: Дата:
                   2663: 01-Nov-2015
                   2664: 
                   2665: Инфо:
                   2666: Команда благодарит за тестирование пакета и ценные замечания Игоря
                   2667: Биткина (Москва), Елизавету Вахранёву (Москва), Галину Воловник
                   2668: (Рыбница), Владимира Городецкого (Иерусалим), Якова Зайдельмана
                   2669: (Переяславль-Залесский), Ирину Зубкову (Нижний Новгород), Александра
                   2670: Карчевского (Раменское), Дмитрия Когана (Мюнхен), Андрея Кокуленко
                   2671: (Омск), Валерия Леонченко (Кишинев), Сергея Лобачёва (Нижний Новгород),
                   2672: Анастасия Стебалину (Москва), Антона Тахтарова (Самара), Ульяну
                   2673: Фабричнину (Санкт-Петербург), а также команду "Сборная Кирибати" за
                   2674: место в высшей лиге и необходимость подготовки пакета.
                   2675: 
                   2676: Вопрос 1:
                   2677: После победного матча за Суперкубок Португалии футболист Ислам Слимани
                   2678: побежал к скамейке запасных, схватил бутылку с водой и опорожнил ее на
                   2679: себя. Что произошло со Слимани непосредственно перед этим?
                   2680: 
                   2681: Ответ:
                   2682: Его облили шампанским.
                   2683: 
                   2684: Зачет:
                   2685: Синонимичные ответы.
                   2686: 
                   2687: Комментарий:
                   2688: Запасной вратарь Марселу Бук обильно облил его праздничным шампанским.
                   2689: Алжирец, как правоверный мусульманин, не приемлет алкоголь в любом виде.
                   2690: Форвард тотчас побежал к скамейке запасных, схватил бутылку с водой и
                   2691: вылил ее на себя, чтобы "обмыться".
                   2692: 
                   2693: Источник:
                   2694: http://lenta.ru/news/2015/08/10/slimani/
                   2695: 
                   2696: Автор:
                   2697: Ольга Кузьма, Андрей Кузьма
                   2698: 
                   2699: Вопрос 2:
                   2700: ИКС, получивший имя в честь ИКСА, был построен только в 1928 году. Какое
                   2701: слово мы заменили ИКСОМ?
                   2702: 
                   2703: Ответ:
                   2704: Цеппелин.
                   2705: 
                   2706: Комментарий:
                   2707: Дирижабль "Граф Цеппелин" был построен уже после смерти графа фон
                   2708: Цеппелина и был далеко не первым цеппелином.
                   2709: 
                   2710: Источник:
                   2711: http://ru.wikipedia.org/wiki/Граф_Цеппелин_(дирижабль)
                   2712: 
                   2713: Автор:
                   2714: Дмитрий Петров
                   2715: 
                   2716: Вопрос 3:
                   2717: Назовите святого, являющегося героем статьи, из которой авторы вопроса
                   2718: узнали о том, что в устье реки Сиговицы было много подземных ключей.
                   2719: 
                   2720: Ответ:
                   2721: Александр Невский.
                   2722: 
                   2723: Зачет:
                   2724: Александр Ярославич.
                   2725: 
                   2726: Комментарий:
                   2727: Из-за ключей в месте впадения Сиговицы в Чудское озеро лед был рыхлым и
                   2728: хрупким.
                   2729: 
                   2730: Источник:
                   2731: А.И. Ермаков. Великие полководцы. 100 историй о подвигах и победах. -
                   2732: М.: Центрполиграф, 2011 - С. 14.
                   2733: 
                   2734: Автор:
                   2735: Ольга Кузьма, Андрей Кузьма
                   2736: 
                   2737: Вопрос 4:
                   2738: На склоне лет Марк Твен писал, что романтика речного кораблевождения
1.2     ! rubashki 2739: умерла, и лоцман уже не ПРОПУСК. Под определение "ПРОПУСК" подходит, в
        !          2740: том числе, Баал. Какие два слова мы дважды пропустили?
1.1       rubashki 2741: 
                   2742: Ответ:
                   2743: Бог Ганнибала.
                   2744: 
                   2745: Комментарий:
                   2746: Детство и юность Марк Твен провел в городе Ганнибал. В те годы многие
                   2747: местные молодые люди, как и сам Марк Твен, подражали речным морякам и
                   2748: мечтали стать лоцманами. Верховными божествами карфагенян был бог Баал и
                   2749: богиня Танит.
                   2750: 
                   2751: Источник:
                   2752:    1. М. Твен. Из записных книжек. 1865-1905.
                   2753: http://www.flibusta.net/b/131699/read
                   2754:    2. http://ru.wikipedia.org/wiki/Карфаген
                   2755: 
                   2756: Автор:
                   2757: Дмитрий Петров
                   2758: 
                   2759: Вопрос 5:
                   2760: Описание последних дней жизни Александра I оставил его лейб-медик
                   2761: Виллие. Некоторые считают это описание недостоверным, ссылаясь на
                   2762: диагноз, который ранее поставил Виллие. Назовите этот диагноз двумя
                   2763: словами.
                   2764: 
                   2765: Ответ:
                   2766: Апоплексический удар.
                   2767: 
                   2768: Комментарий:
                   2769: Данный диагноз был поставлен Павлу I.
                   2770: 
                   2771: Источник:
                   2772: В.М. Скляренко, В.В. Сядро, О.Ю. Очкурова. 50 знаменитых загадок истории
                   2773: XVIII-XIX веков. - Харьков: Фолио, 2013. - С. 376.
                   2774: 
                   2775: Автор:
                   2776: Ольга Кузьма, Андрей Кузьма
                   2777: 
                   2778: Вопрос 6:
                   2779: [Ведущему: ле-кон-Ария.]
                   2780:    Алан Паркер говорит, что, когда Ленин жил в северном Лондоне, ПЕРВЫЕ
                   2781: еще не переехали. Поэтому Ленин, скорее всего, поддерживал ВТОРЫХ. У
                   2782: ПЕРВЫХ из ЛеКонАрии были ВТОРЫЕ. Назовите ПЕРВЫХ и ВТОРЫЕ.
                   2783: 
                   2784: Ответ:
                   2785: Канониры, шпоры.
                   2786: 
                   2787: Зачет:
                   2788: Пушкари, шпоры; другие синонимичные ответы.
                   2789: 
                   2790: Комментарий:
                   2791: Ленин, по некоторым сведениям, болел за "Tottenham Hotspur", т.е. за
                   2792: "шпоры", а не за любимый Паркером "Arsenal", т.е. за "канониров".
                   2793: Кстати, "Arsenal" играет в красной (ранее в темно-красной) форме, что
                   2794: тоже могло бы импонировать Ленину. "ЛеКонАрия" - обиходное название
                   2795: корпуса лейб-гвардии конной артиллерии. Вся прислуга конной артиллерии
                   2796: ездила верхом.
                   2797: 
                   2798: Источник:
                   2799:    1. https://esquire.ru/wil/alan-parker
                   2800:    2. http://en.wikipedia.org/wiki/Tottenham_Hotspur_F.C.
                   2801:    3. http://en.wikipedia.org/wiki/Arsenal_F.C.
                   2802:    4. http://ru.wikipedia.org/wiki/Лейб-гвардии_Конная_артиллерия
                   2803:    5. http://dic.academic.ru/dic.nsf/brokgauz_efron/54658/
                   2804: 
                   2805: Автор:
                   2806: Дмитрий Петров
                   2807: 
                   2808: Вопрос 7:
                   2809: Вешалка, висящая над гардеробом в петербургском "Театре за Черной
                   2810: речкой", стилизована под НЕЕ. Другая ОНА пострадала в 1988 году в рамках
                   2811: борьбы с привилегиями. Назовите ЕЕ.
                   2812: 
                   2813: Ответ:
                   2814: Чайка.
                   2815: 
                   2816: Комментарий:
                   2817: В 1988 году, в рамках так называемой "борьбы с привилегиями", автомобиль
                   2818: ГАЗ-14 ("Чайка") был снят с производства по личному указанию М.С.
                   2819: Горбачёва.
                   2820: 
                   2821: Источник:
                   2822:    1. ЛОАВ.
                   2823:    2. http://ru.wikipedia.org/wiki/ГАЗ-14
                   2824: 
                   2825: Автор:
                   2826: Ольга Кузьма, Андрей Кузьма
                   2827: 
                   2828: Вопрос 8:
                   2829: Иван Якубовский вспоминает, что первым догадался СДЕЛАТЬ ЭТО с
                   2830: "Черчиллем". Ответьте одним глаголом, как подобное действие стали
                   2831: называть коллеги Якубовского.
                   2832: 
                   2833: Ответ:
                   2834: Подковать.
                   2835: 
                   2836: Комментарий:
                   2837: Поставлявшийся по ленд-лизу британский танк Mk4 Churchill для улучшения
                   2838: проходимости "подковывали", нанося на гусеницы шипы.
                   2839: 
                   2840: Источник:
                   2841: М.Б. Барятинский. Танки ленд-лиза в бою.
                   2842: http://www.flibusta.net/b/204447/read
                   2843: 
                   2844: Автор:
                   2845: Дмитрий Петров
                   2846: 
                   2847: Вопрос 9:
                   2848: В вопросе словом "ИКС" заменено другое слово.
                   2849:    Юрий Батурин пишет, что сейчас в ИКСАХ, вопреки стереотипу, можно
                   2850: встретить только мед, творог, иногда соки. Назовите автора произведений,
                   2851: один из персонажей которых носит имя ИКС.
                   2852: 
                   2853: Ответ:
                   2854: [Николай Николаевич] Носов.
                   2855: 
                   2856: Комментарий:
                   2857: ИКС - тюбик. В первой части вопроса речь идет о питании космонавтов.
                   2858: 
                   2859: Источник:
                   2860: Ю.М. Батурин. Повседневная жизнь российских космонавтов. - М.: Молодая
                   2861: гвардия, 2011. - С. 199.
                   2862: 
                   2863: Автор:
                   2864: Ольга Кузьма, Андрей Кузьма
                   2865: 
                   2866: Вопрос 10:
                   2867: В Мозамбике для НИХ есть минимальное ограничение по высоте - пять с
                   2868: небольшим метров. По мнению одного исследователя, в известном
                   2869: фольклорном произведении ОНИ подчеркивают гротескность происходящего.
                   2870: Назовите ИХ двумя словами.
                   2871: 
                   2872: Ответ:
                   2873: Высоковольтные провода.
                   2874: 
                   2875: Комментарий:
                   2876: В Мозамбике таким образом проявляют заботу о жирафах. Известное
                   2877: произведение - один из вариантов стихотворения о старушке, которая
                   2878: недолго мучилась.
                   2879: 
                   2880: Источник:
                   2881:    1. http://tdworld.com/overhead-distribution/power-people
                   2882:    2. http://www.ruthenia.ru/apr/textes/klubkov60/belousov.html
                   2883: 
                   2884: Автор:
                   2885: Дмитрий Петров
                   2886: 
                   2887: Вопрос 11:
                   2888: Одна из работ на выставке, посвященной Великому шелковому пути,
                   2889: представляла собой ПЕРВОГО с рядами ВТОРЫХ по бокам. Какие слова,
                   2890: начинающиеся на одну и ту же букву, мы заменили на "ПЕРВЫЙ" и "ВТОРЫЕ"?
                   2891: 
                   2892: Ответ:
                   2893: Верблюд, весла.
                   2894: 
                   2895: Комментарий:
                   2896: Верблюд - корабль пустыни.
                   2897: 
                   2898: Источник:
                   2899: Наблюдения авторов вопроса на выставке "Императорские сады", 08.06.2015
                   2900: г.
                   2901: 
                   2902: Автор:
                   2903: Ольга Кузьма, Андрей Кузьма
                   2904: 
                   2905: Вопрос 12:
                   2906: В сонете, посвященном работе над более известным произведением того же
                   2907: автора, есть строка "А с кисти на лицо течет бурда". Назовите автора.
                   2908: 
                   2909: Ответ:
                   2910: Микеланджело [Буонаротти].
                   2911: 
                   2912: Комментарий:
                   2913: Микеланджело расписывал потолок Сикстинской капеллы лежа, иногда - в не
                   2914: самом удобном положении.
                   2915: 
                   2916: Источник:
                   2917: Р. Хьюз. Рим. История города: его культура, облик, люди. - М.: АСТ:
                   2918: CORPUS, 2014. - С. 251.
                   2919: 
                   2920: Автор:
                   2921: Дмитрий Петров
                   2922: 
                   2923: Вопрос 13:
                   2924: Рассказывая об охватившей артисток балета моде на исполнение фуэте,
                   2925: Ольга Ковалик использует слово персидского происхождения. Напишите его.
                   2926: 
                   2927: Ответ:
                   2928: Дервиш.
                   2929: 
                   2930: Комментарий:
                   2931: "Училище превратилось в скопище кружащихся дервишей".
                   2932: 
                   2933: Источник:
                   2934: О.Г. Ковалик. Повседневная жизнь балерин русского императорского театра.
                   2935: - М.: Молодая гвардия, 2011. - С. 94.
                   2936: 
                   2937: Автор:
                   2938: Ольга Кузьма, Андрей Кузьма
                   2939: 
                   2940: Вопрос 14:
                   2941: Соотечественница сравнила ЕГО с Фениксом, родившимся из огня и пепла. В
                   2942: одном ЕГО англоязычном произведении встречается слово "Backfisch"
                   2943: [бакфИш], означающее в том числе мелкую рыбешку, пригодную только для
                   2944: выпечки. Назовите это произведение.
                   2945: 
                   2946: Ответ:
                   2947: "Лолита".
                   2948: 
                   2949: Комментарий:
                   2950: Нина Берберова писала, что Набоков как писатель, подобно Фениксу,
                   2951: родился из огня и пепла революции и изгнания. Владимир Набоков в начале
                   2952: писательской карьеры публиковался под псевдонимом Сирин. Сирин, как и
                   2953: Феникс, - мифическая птица. В немецком слово "Backfisch" [бакфИш] также
                   2954: обозначает девочку на определенной стадии взросления.
                   2955: 
                   2956: Источник:
                   2957:    1. Н.Н. Берберова. Курсив мой. http://www.flibusta.net/b/181192/read
                   2958:    2. https://lilithmaud.wordpress.com/2013/04/16/nabokovs-lolita-and-its-more-obscure-vocabulary/
                   2959:    3. http://de.wikipedia.org/wiki/Backfisch_(M%C3%A4dchen)
                   2960: 
                   2961: Автор:
                   2962: Дмитрий Петров
                   2963: 
                   2964: Вопрос 15:
                   2965: Готовясь подать заявку на участие в Чемпионате Санкт-Петербурга - 2015,
                   2966: авторы вопроса узнали, что Хайнц Гюнтхард стал первым человеком,
                   2967: выигравшим теннисный турнир, являясь ИМ. Назовите ЕГО двумя словами.
                   2968: 
                   2969: Ответ:
                   2970: Lucky looser.
                   2971: 
                   2972: Зачет:
                   2973: Лаки лузер; счастливый неудачник.
                   2974: 
                   2975: Комментарий:
                   2976: Команда "Ноев ковчег" хотела выбрать для чемпионата города название,
                   2977: отражающее тот факт, что по итогам предыдущего чемпионата не завоевала
                   2978: место в высшей лиге. Термин "лаки лузер" означает спортсмена,
                   2979: включенного в основной список участников турнира вместо выбывшего в
                   2980: последний момент.
                   2981: 
                   2982: Источник:
                   2983: http://ru.wikipedia.org/wiki/Гюнтхардт,_Хайнц
                   2984: 
                   2985: Автор:
                   2986: Ольга Кузьма, Андрей Кузьма
                   2987: 
                   2988: Тур:
10 тур. "Анабасис-вектор"
                   2989: 
                   2990: Дата:
                   2991: 01-Nov-2015
                   2992: 
                   2993: Редактор:
                   2994: Алексей Тыртычный
                   2995: 
                   2996: Инфо:
                   2997: Команда благодарит координатора турнира Владимира Григорьевича Белкина
                   2998: за ценные советы.
                   2999: 
                   3000: Вопрос 1:
                   3001: Мы надеемся, что даже если тур нашей команды вам не понравится, то вы
                   3002: всё же узнаете новые интересные факты, что подтолкнет вас в дальнейшем
                   3003: играть лучше.
                   3004:    В одной из песен российской группы фигурирует исторический персонаж,
                   3005: живший до нашей эры. Далее в тексте упоминается плеть, смирительная
                   3006: рубашка и звучит "Яволь". Назовите имя этого персонажа.
                   3007: 
                   3008: Ответ:
                   3009: Заратустра.
                   3010: 
                   3011: Комментарий:
                   3012: Мы решили начать наш тур так же, как начинается телевизионная версия
                   3013: "Что? Где? Когда?". Песня "Заратустра" есть и у группы "Пикник". Судя по
                   3014: тому, что в тексте фигурирует плеть, немецкое слово "Яволь", а также
                   3015: фраза "Не таков он, Заратустра, чтобы каждого жалеть", автора песни
                   3016: вдохновило произведение Фридриха Ницше "Так говорил Заратустра".
                   3017: Известна фраза Ницше "Ты идешь к женщинам? Не забудь плетку". Ницше
                   3018: закончил свои дни в сумасшедшем доме. Вступительное обращение к командам
                   3019: было аллюзией на другую известную цитату из Ницше: "Что нас не убивает,
                   3020: делает нас сильнее", а слово "подтолкнуть" могло напомнить о еще одной
                   3021: его фразе - "Что падает, то нужно еще толкнуть".
                   3022: 
                   3023: Источник:
                   3024:    1. http://megalyrics.ru/lyric/piknik/zaratustra.htm
                   3025:    2. Ф. Ницше. Так говорил Заратустра.
                   3026: http://www.flibusta.net/b/406494/read
                   3027: 
                   3028: Автор:
                   3029: Ренарт Фасхутдинов
                   3030: 
                   3031: Вопрос 2:
                   3032: В книге "Под пологом пьяного леса" Джеральд Даррелл сравнивает клювы
                   3033: маленьких совят с тем, что вопреки названию чаще изготавливают из
                   3034: грецкого ореха, самшита, палисандра. С чем же сравниваются автором клювы
                   3035: совят?
                   3036: 
                   3037: Ответ:
                   3038: Кастаньеты.
                   3039: 
                   3040: Комментарий:
                   3041: Выпрашивающие пищу совята щелкали своими клювами подобно кастаньетам.
                   3042: Испанское слово "casta&ntilde;a" [кастАнья] означает "каштан".
                   3043: 
                   3044: Источник:
                   3045:    1. Дж. Даррелл. Под пологом пьяного леса.
                   3046: http://www.flibusta.net/b/12071/read
                   3047:    2. http://www.pianinotrade.ru/en-muz-instr/kastanety
                   3048: 
                   3049: Автор:
                   3050: Алексей Тыртычный
                   3051: 
                   3052: Вопрос 3:
                   3053: (pic: 20150712.jpg)
                   3054:    В 1991 году в журнале "Трамвай" была опубликована игра по мотивам
                   3055: известного литературного произведения. Вы видите игровое поле, в центре
                   3056: которого расположена фишка. Задача игроков - бросая кубик, переместить
                   3057: эту фишку на свою сторону поля. Ответьте максимально точно одним словом,
                   3058: какой объект символизирует эта фишка.
                   3059: 
                   3060: Ответ:
                   3061: Воз.
                   3062: 
                   3063: Комментарий:
                   3064: Игра по мотивам басни Крылова.
                   3065: 
                   3066: Источник:
                   3067: Журнал "Трамвай", 1991, N 10.
                   3068: 
                   3069: Автор:
                   3070: Виталий Баринов
                   3071: 
                   3072: Вопрос 4:
                   3073: (pic: 20150713.jpg)
                   3074:    Перед вами Джейме и Серсея Ланнистеры - любящие брат и сестра из
                   3075: сериала "Игра Престолов". На сайте pikabu.ru [пикабу ру] этот кадр
                   3076: характеризуется двухкоренным неологизмом, часть которого составляет
                   3077: название сериала, вышедшего в 2004 году. Воспроизведите этот неологизм.
                   3078: 
                   3079: Ответ:
                   3080: Кровосмешарики.
                   3081: 
                   3082: Комментарий:
                   3083: (pic: 20150714.jpg)
                   3084: 
                   3085: Источник:
                   3086:    1. http://pikabu.ru/story/krovosmeshariki_2422825
                   3087:    2. http://ru.wikipedia.org/wiki/Смешарики
                   3088: 
                   3089: Автор:
                   3090: Николай Московой
                   3091: 
                   3092: Вопрос 5:
                   3093: [Ведущему: кавычки не озвучивать.]
                   3094:    Он собирался в Индию, но не нашел "Шотландию" и в итоге отправился
                   3095: искать "Британию". Он не смог вернуться во Францию из-за боязни навлечь
                   3096: насмешки на общество, в котором состоял. Назовите причину этой боязни
                   3097: одним десятибуквенным словом.
                   3098: 
                   3099: Ответ:
                   3100: Татуировка.
                   3101: 
                   3102: Зачет:
                   3103: Татуировки.
                   3104: 
                   3105: Комментарий:
                   3106: Сев вместо судна "Шотландия" на корабль "Дункан", Паганель направился не
                   3107: в Индию, а на поиски капитана Гранта и его матросов, потерпевших
                   3108: крушение на корабле "Британия". Сделанными у маорийцев татуировками
                   3109: Паганель боялся навлечь насмешки на Географическое общество.
                   3110: 
                   3111: Источник:
                   3112: Ж. Верн. Дети капитана Гранта. http://www.flibusta.net/b/400084/read
                   3113: 
                   3114: Автор:
                   3115: Алексей Тыртычный
                   3116: 
                   3117: Вопрос 6:
                   3118: Этого литературного персонажа на его родном языке звали Трахальд, что
                   3119: можно перевести как "роющий нору, ввинчивающийся вглубь". Из двух же
                   3120: имен, под которыми он нам больше известен, одно происходит из
                   3121: древнеанглийского языка, а другое, предположительно, - из
                   3122: древненорвежского. Назовите любое из этих имен.
                   3123: 
                   3124: Ответ:
                   3125: Голлум.
                   3126: 
                   3127: Зачет:
                   3128: Смеагол; Горлум; Смеагорл; Gollum; Smeagol.
                   3129: 
                   3130: Комментарий:
                   3131: Вопрос легко можно было взять, если в вашей команде кто-то знает
                   3132: старонорвежский язык. :-) Трахальд - имя персонажа на придуманном
                   3133: Толкином языке. В основном тексте "Властелина колец" этот язык передан
                   3134: английским.
                   3135: 
                   3136: Источник:
                   3137:    1. Дж.Р.Р. Толкин. Руководство по переводу имен собственных из
                   3138: "Властелина Колец". http://www.flibusta.net/b/167956/read
                   3139:    2. http://ru.wikipedia.org/wiki/Голлум
                   3140: 
                   3141: Автор:
                   3142: Виталий Баринов
                   3143: 
                   3144: Вопрос 7:
                   3145: Известный знаток Нурали Латыпов последние годы занимается вопросами
                   3146: интеллектуального развития человека. В своей книге, вышедшей в 2014
                   3147: году, он приводит несколько десятков упражнений для развития мозга и
                   3148: поддержания его в должной интеллектуальной форме. Каким существующим
                   3149: только во множественном числе словом называет Латыпов эти упражнения?
                   3150: 
                   3151: Ответ:
                   3152: Бигуди.
                   3153: 
                   3154: Комментарий:
                   3155: Характерную форму головному мозгу придают так называемые извилины.
                   3156: Считается, чем больше в мозге извилин, тем человек умнее. Приводимые в
                   3157: книге задачи и упражнения Латыпов рассматривает как своеобразные бигуди
                   3158: для извилин.
                   3159: 
                   3160: Источник:
                   3161: Н.Н. Латыпов. Бигуди для извилин. Возьми от мозга всё! - М.: АСТ, 2014.
                   3162: 
                   3163: Автор:
                   3164: Александр Скородумов
                   3165: 
                   3166: Вопрос 8:
                   3167: Внимание, в вопросе есть замены.
                   3168:    Французский энтомолог Жан-Анри Фабр сетовал, что, в то время как он
                   3169: часами наблюдал за насекомыми, стараясь определить наличие у них ИКСА
                   3170: или ИГРЕКА, случайные прохожие приходили к выводу, что ИГРЕКА нет у него
                   3171: самого. Ответьте одним словом, какой ИКС упоминается в названии фильма
                   3172: 1992 года.
                   3173: 
                   3174: Ответ:
                   3175: Основной.
                   3176: 
                   3177: Комментарий:
                   3178: ИКС - инстинкт, ИГРЕК - разум. Прохожие принимали Фабра, часами
                   3179: наблюдавшего насекомых, за сумасшедшего.
                   3180: 
                   3181: Источник:
                   3182:    1. Ж.-А. Фабр. Жизнь насекомых. Рассказы энтомолога.
                   3183: http://www.flibusta.net/b/308146/read
                   3184:    2. http://ru.wikipedia.org/wiki/Основной_инстинкт_(фильм)
                   3185: 
                   3186: Автор:
                   3187: Виталий Баринов
                   3188: 
                   3189: Вопрос 9:
                   3190: Внимание, в вопросе словом "ОНИ" заменено другое слово.
                   3191:    В самом начале кинофильма "Социальная сеть" главный герой Марк
                   3192: Цукерберг заявляет, что ОНИ ему не нужны. Ответом на какую стандартную
                   3193: фразу его собеседницы служит это заявление Цукерберга?
                   3194: 
                   3195: Ответ:
                   3196: "Нам лучше остаться друзьями".
                   3197: 
                   3198: Зачет:
                   3199: "Будем друзьями" и иные фразы с сохранением данного смысла и упоминанием
                   3200: друзей.
                   3201: 
                   3202: Комментарий:
                   3203: В самом начале фильма никому еще не известный студент Марк Цукерберг
                   3204: пытается ухаживать за девушкой, но она отказывает ему со словами "Нам
                   3205: лучше остаться друзьями". "Мне не нужны друзья", - отвечает Цукерберг. А
                   3206: скоро он создает свою первую университетскую сеть, и у него появляется
                   3207: много-много друзей.
                   3208: 
                   3209: Источник:
                   3210: Фильм "Социальная сеть" (2010), реж. Дэвид Финчер.
                   3211: 
                   3212: Автор:
                   3213: Александр Скородумов
                   3214: 
                   3215: Вопрос 10:
                   3216: Вопрос посвящается команде "Эклипс", тур которой в чемпионате нам очень
                   3217: понравился. Не ищите в этом какую-то подсказку на ответ.
                   3218:    Действие романа Фейхтвангера "Еврейка из Толедо" происходит в
                   3219: средневековой Испании времен Альфонса VIII. Автору вопроса показалось
                   3220: странным, что в театральной постановке этого романа эксплуатировался
                   3221: один из объектов Всемирного наследия, которому этот статус был присужден
                   3222: ЮНЕСКО в 2010 году. Назовите этот "огненный" объект Всемирного наследия
                   3223: одним словом.
                   3224: 
                   3225: Ответ:
                   3226: Фламенко.
                   3227: 
                   3228: Комментарий:
                   3229: По различным источникам, фламенко появилось не ранее XV века. 16 ноября
                   3230: 2010 года ЮНЕСКО присудила фламенко статус объекта Всемирного Наследия.
                   3231: 
                   3232: Источник:
                   3233:    1. http://ru.wikipedia.org/wiki/Фламенко
                   3234:    2. Спектакль "Испанская баллада" в Театре им. Ленсовета в
                   3235: Санкт-Петербурге.
                   3236: 
                   3237: Автор:
                   3238: Алексей Тыртычный
                   3239: 
                   3240: Вопрос 11:
                   3241: Без Шекспира наш пакет мы также решили не оставлять.
                   3242:    Одну из разновидностей ИХ - центифОлию - называют также капустной.
                   3243: Другую ранние христиане связывали с Девой Марией, еще одну - с кровью
                   3244: Христа. В выдуманной Шекспиром сцене в одной из его пьес представлен
                   3245: процесс выбора из двух последних упомянутых выше разновидностей ИХ.
                   3246: Назовите в правильном порядке тех, кто делал этот выбор.
                   3247: 
                   3248: Ответ:
                   3249: Йорки, Ланкастеры.
                   3250: 
                   3251: Зачет:
                   3252: По фамилиям.
                   3253: 
                   3254: Комментарий:
                   3255: Столистную розу, или центифОлию, называют также капустной розой. Белые
                   3256: розы - символ Девы Марии, алые - крови Христа. Речь в вопросе идет об
                   3257: участниках "Войны роз" в Англии.
                   3258: 
                   3259: Источник:
                   3260:    1. http://i-fakt.ru/interesnye-fakty-o-rozax/
                   3261:    2. http://www.инфо-сад.рф/publ/rozy_vidy_roz_prodolzhenie/1-1-0-18
                   3262:    3. http://ru.wikipedia.org/wiki/Война_Алой_и_Белой_розы
                   3263: 
                   3264: Автор:
                   3265: Алексей Тыртычный
                   3266: 
                   3267: Вопрос 12:
                   3268: Внимание, в вопросе присутствуют замены.
                   3269:    В середине пятнадцатого века правитель Османской империи присвоил
                   3270: себе титул, который буквально означает "ИКС ИГРЕКА". Известный ИКС,
                   3271: будущий государь, родился в 1475 году в ИГРЕКЕ. Назовите фамилию этого
                   3272: ИКСА.
                   3273: 
                   3274: Ответ:
                   3275: Борджиа.
                   3276: 
                   3277: Комментарий:
                   3278: В 1453 году османский султан Мехмед II захватил Константинополь, после
                   3279: чего взял себе титул Кайзер-и-Рум, что буквально означает "Цезарь Рима".
                   3280: Поводом к этому послужило то, что официальное название Константинополя
                   3281: было Новый Рим, а сама Византия именовалась Ромея, т.е. Римская империя.
                   3282: В 1475 году в Риме родился Цезарь Борджиа (в итальянской огласовке -
                   3283: Чезаре), будущий правитель Романьи. Его образ использован Макиавелли в
                   3284: трактате "Государь".
                   3285: 
                   3286: Источник:
                   3287:    1. http://ru.wikipedia.org/wiki/Падение_Константинополя_(1453)
                   3288:    2. http://ru.wikipedia.org/wiki/Борджиа,_Чезаре
                   3289: 
                   3290: Автор:
                   3291: Ренарт Фасхутдинов
                   3292: 
                   3293: Вопрос 13:
                   3294: Одна из статей на сайте "Российской газеты" описывает препятствия для
                   3295: экономического роста России, которые создает увеличение цен на
                   3296: электрическую энергию. Название этой статьи на две буквы длиннее
                   3297: названия природного объекта Всемирного наследия, на котором встречаются
                   3298: коровы. Воспроизведите название статьи, состоящее из трех слов.
                   3299: 
                   3300: Ответ:
                   3301: "Большой Барьерный Тариф".
                   3302: 
                   3303: Комментарий:
                   3304: На Большом Барьерном рифе встречаются морские коровы.
                   3305: 
                   3306: Источник:
                   3307:    1. http://www.rg.ru/2011/03/14/energetika.html
                   3308:    2. http://ru.wikipedia.org/wiki/Большой_Барьерный_риф
                   3309:    3. http://www.destinations.ru/fact/52
                   3310: 
                   3311: Автор:
                   3312: Алексей Тыртычный
                   3313: 
                   3314: Вопрос 14:
                   3315:    <раздатка>
                   3316:    ... мистрис Гейз... подошла и снисходительно сказала: "А вы просто
                   3317: шлепните ее хорошенько, если она вам мешает в ваших размышлениях. Как я
                   3318: люблю этот сад", - продолжала она без ПРОПУСК1. - "А это солнце, разве
                   3319: это не рай (ПРОПУСК2 тоже отсутствует)".
                   3320:    </раздатка>
                   3321:    Ответьте в правильном порядке (оба раза - двумя словами), что в
                   3322: полученном вами фрагменте из романа Набокова "Лолита" было заменено на
                   3323: ПРОПУСК1, а что - на ПРОПУСК2.
                   3324: 
                   3325: Ответ:
                   3326: "... восклицательного знака...", "... вопросительный знак...".
                   3327: 
                   3328: Комментарий:
                   3329: Мы не знаем, какие эмоции в большей степени вызвал наш тур -
                   3330: вопросительные или восклицательные, но надеемся, что он не был воспринят
                   3331: вами равнодушно по принципам мистрис Гейз.
                   3332: 
                   3333: Источник:
                   3334: В.В. Набоков. Лолита. http://www.flibusta.net/b/425308/read
                   3335: 
                   3336: Автор:
                   3337: Алексей Тыртычный
                   3338: 
                   3339: Вопрос 15:
                   3340: [Ведущему: не реагировать на поправки из зала, текст вопроса прочитать
                   3341: медленно и отчетливо, но отдельные слова не повторять.]
                   3342:    <раздатка>
                   3343:    Догадавшись отсутствие чего в некоторых собственных статьях удивляло
                   3344: героя Чопека, назовите латинским словом изобретение венерианского
                   3345: издателя Габриэля Пьерри.
                   3346:    </раздатка>
                   3347:    Догадавшись, отсутствие чего в некоторых собственных статьях удивляло
                   3348: героя Чапека, назовите латинским словом изобретение венецианского
                   3349: издателя Габриэля Пьерри.
                   3350: 
                   3351: Ответ:
                   3352: Errata.
                   3353: 
                   3354: Зачет:
                   3355: Erratum; эррата; эрратум.
                   3356: 
                   3357: Комментарий:
                   3358: Как вы могли видеть, в полученном вами тексте вопроса присутствовали
                   3359: опечатки. Героя Чапека удивляло, что в некоторых его статьях опечаток не
                   3360: было. Габриэль Пьерри впервые стал помещать в конце книги список
                   3361: замеченных опечаток - эррату.
                   3362:    Надеемся, наш тур вам понравился, мы учтем замечания и постараемся не
                   3363: допускать отмеченных вами ошибок.
                   3364: 
                   3365: Источник:
                   3366:    1. К. Чапек. Как делается газета.
                   3367: http://www.flibusta.net/b/82290/read
                   3368:    2. Д.Ю. Шерих. "А" упало, "Б" пропало... Занимательная история
                   3369: опечаток. http://www.flibusta.net/b/249148/read
                   3370: 
                   3371: Автор:
                   3372: Алексей Тыртычный
                   3373: 

FreeBSD-CVSweb <freebsd-cvsweb@FreeBSD.org>